Archer Respiratory

Pataasin ang iyong marka sa homework at exams ngayon gamit ang Quizwiz!

The nurse attends to a client with shortness of breath, bilateral lung crackles, weak pulses, and frothy pink sputum. Which of the following orders should the nurse question for this client? A. Supplemental oxygen via nasal cannula or mask B. Losartan C. Fowler's position D. Diltiazem

Explanation Choice D is correct. The client is exhibiting symptoms and signs of left ventricular heart failure. Decreased cardiac output associated with acute systolic heart failure results in reduced blood pressure, weak pulses, and acute pulmonary edema ( dyspnea, frothy pink sputum, lung crackles). Diltiazem and other calcium channel blockers ( CCBs) produce a negative inotropic effect ( reduced myocardial contractility) and are contraindicated in acute systolic heart failure. CCBs may exacerbate systolic dysfunction and cause heart failure symptoms to worsen. The nurse should question this order to determine if there is a more appropriate medication to accomplish the intended therapeutic effect with a lower risk of complications. Choice A, B, and C are incorrect. These orders are appropriate for a client presenting with acute heart failure and pulmonary edema. The nurse should administer supplemental oxygen to promote adequate tissue oxygenation ( Choice A). Losartan is an angiotensin receptor blocker ( ARV). Afterload is increased in systolic heart failure due to RAAS ( Renin-Angiotensin- Aldosterone-System) mediated increased peripheral vascular resistance. By blocking angiotensin II receptors, losartan causes vasodilation and decreases afterload. Losartan is indicated in systolic heart failure because it reduces mortality and morbidity. Fowler's position ( Choice C) is indicated for patients with heart failure symptoms. Fowler's position promotes oxygenation by allowing maximum chest expansion. NCSBN Client Need Topic: Adult health - Cardiovascular, Subtopic: Illness management, the potential for alterations in body systems, system-specific assessments, expected actions/outcomes Learning Objective Recognize acute heart failure symptoms and understand that the drugs that reduce myocardial contractility ( negative inotropics) are contraindicated in this setting. Additional Info Heart failure is associated with reduced cardiac output and reduced blood flow to organs, including the kidneys. Reduced renal blood flow stimulates renin release. Renin converts Angiotensinogen to Angiotensin I, which is further activated to Angiotensin II by the angiotensin-converting enzyme in the lungs. Angiotensin II is a vasoconstrictor, and it increases peripheral vascular resistance (afterload). When medications are used to reduce afterload, the heart pumps better and cardiac output increases. Losartan is an angiotensin receptor blocker ( ARB). It reduces the systemic blood pressure (afterload) by countering the angiotensin II. Losartan does not have direct inotropic action on the heart, but it helps the cardiac output by decreasing the afterload. Losartan improves the morbidity and mortality in heart failure, and hence it's an important drug in treating heart failure.

The nurse reviews prescriptions for assigned clients. Which prescription should the nurse question? A. Albuterol via nebulizer for a client with hypokalemia. B. Clozapine for a client with severe schizophrenia. C. Lisinopril for a client with congestive heart failure. D. Verapamil for a client with migraine headaches. Submit Answer

Explanation Choice A is correct. Albuterol is a bronchodilator that is used for asthma exacerbations. Adversely, this medication may lower serum potassium levels. The nurse should question this order as this medication may decrease the potassium further. Choices B, C, and D are incorrect. Clozapine is an atypical antipsychotic used in the treatment of severe schizophrenia. Lisinopril is an ACE inhibitor indicated for heart failure and hypertension. Verapamil is commonly used as prophylaxis for migraine headaches; this medication may also be used for individuals with hypertension. Learning Objective Recognize that albuterol can drive potassium into the cells and cause hypokalemia. Additional Info Albuterol is a short-acting bronchodilator. Common side effects of albuterol include tremors, tachycardia, palpitations, and metabolic disturbances such as hypokalemia and hyperglycemia. This medication is emergently indicated for asthma exacerbations. Sometimes, it is used as an adjunctive agent for treating hyperkalemia because it can drive potassium into the cells and lower serum potassium.

A client who sustained a motor vehicle accident developed pneumothorax and a chest tube was inserted to treat this. After several days, the pneumothorax was relieved and the physician ordered the chest tube removal. Which intervention is most important before removing the tube? A. Administration of premedication 1 hour before the procedure to ensure client's comfort. B. Prepare 1 unit of packed RBCs for transfusion. C. Clamp the tube 2 hours before the removal. D. Ask the client to sign the consent form for removal. Submit Answer

Explanation Choice A is correct. An analgesic is given 1 hour before the removal of the chest tube. The nurse should wait until the medication has taken effect before the procedure is performed. Choices B, C, and D are incorrect. There is no need for a blood transfusion for this procedure. Clamping of the tube is not done unless ordered by the physician. The consent is usually given when the chest tube was inserted. Additional Info Last Updated - 02, Dec 2021

The clinic nurse is providing teaching to a patient who has been newly prescribed a corticosteroid inhaler. Which of the following information should be included in this teaching? A. Push the canister on the inhaler down, then breathe in slowly and deeply before exhaling B. Do not use a spacer with this medication C. You will not need to rinse your mouth after using this inhaler D. Inhale quickly to prevent the medicine from escaping through the mouth

Explanation Choice A is correct. Breathing in slowly and deeply while using an inhaled medication is the proper method of medication administration. Choice B is incorrect. Spacers are encouraged because they prevent the accumulation of the medication at the back of the throat. Choice C is incorrect. The patient should rinse their mouth after the administration of this medication to prevent Candida infections. Choice D is incorrect. Inhaling quickly is not the best way to take this medication. NCSBN client need Topic: Physiological Integrity, Pharmacological and parenteral therapies Last Updated - 01, Feb 2022

The nurse is educating a client scheduled for pulmonary function tests. It would indicate effective teaching if the client makes which statement? A. "I should not use my bronchodilator four to six hours before these tests." B. "I should not eat or drink six to eight hours prior to these tests." C. "I will need someone to drive me home after I wake up from the anesthesia." D. "My gag reflex will have to return before I resume eating and drinking." Submit Answer

Explanation Choice A is correct. Bronchodilator medications are withheld four to six hours prior to the testing. The purpose of holding this medication is to prevent skewing of the results. Choices B, C, and D are incorrect. These statements are incorrect and require follow-up. PFTs do not require a client to be NPO and are non-invasive, so anesthesia is not utilized. PFTs may be performed at the bedside. Additional Info Pulmonary function tests (PFTs) do not require any sedation or invasive machinery and may be done at the bedside. The purpose is to assess lung function and breathing problems. These tests measure lung volumes and capacities, flow rates, diffusion capacity, gas exchange, airway resistance, and ventilation distribution. The results are interpreted by comparing the patient's data with expected findings for age, gender, race, height, weight, and smoking status. Before the testing, the client is instructed to withhold any bronchodilators four to six hours prior, abstain from smoking, and refrain from wearing tight or restrictive clothing.

A client was admitted for acute exacerbation of asthma. Auscultation findings reveal almost absent breath sounds. Albuterol nebulization was administered. Thirty minutes later, the nurse auscultates and hears diffuse inspiratory and expiratory wheezes throughout the lung fields. This finding means: A. There is increased airflow B. There is no improvement in the airflow C. There is worsening of the condition D. The airflow issue was not addressed Submit Answer

Explanation Choice A is correct. Changes in breath sounds indicate that the client has responded to treatment. When the client came in, there were nearly absent breath sounds, indicating that there was severe airflow obstruction. A noisy chest is a sign that airflow has improved even though they are still partially obstructed. Choices B, C, and D are incorrect. Additional Info Source : Archer Review Last Updated - 12, Oct 2021

The nurse performs a respiratory assessment on an older adult client and observes apnea alternates with periods of rapid breathing. The nurse should document this respiratory pattern as A. Cheyne-Stokes. B. Kussmaul's. C. agonal. D. tachypnea. Submit Answer

Explanation Choice A is correct. Cheyne-Stokes respirations are characterized when the respiratory rate and depth are irregular and alternate with periods of apnea and hyperventilation. The pattern then reverses, and the breathing slows and becomes shallow, concluding as apnea before respiration resumes. This may occur in older adults without any underlying pathology. This also could be characteristic of the client's critical condition following a neurological injury. Choices B, C, and D are incorrect. ➢ Kussmaul's are characterized by respirations that are abnormally deep, regular, and increased in rate ➢ Agonal breathing is always concerning because the client is gasping for air and warrants immediate intervention ➢ Tachypnea is when the respiratory rate is > 20 respirations per minute and is associated with conditions such as fear, anxiety, etc Additional Info Source: Archer ReviewSource: Archer Review Last Updated - 04, Jan 2023

The nurse is administering medications to a 5-year-old client diagnosed with pneumonia. The health care provider has ordered a cough suppressant. Which medication does the nurse administer? A. Dextromethorphan B. Guaifenesin C. Dexmedetomidine D. Protonix Submit Answer

Explanation Choice A is correct. Dextromethorphan is a cough suppressant. It is the ingredient in many over the counter cough medicines such as Delsym, Robitussin, and NyQuil. Dextromethorphan works by signaling the brain to stop triggering the cough reflex. Choice B is incorrect. Guaifenesin is an expectorant, not a cough suppressant. Unlike a cough suppressant, guaifenesin loosens the congestion in a client's chest and throat making it easier for them to cough out mucus and phlegm. Choice C is incorrect. Dexmedetomidine is a sedative medication. It activates receptors in the brain that inhibit neuronal firing, which causes sedation. It is not a cough suppressant. Choice D is incorrect. Protonix is a proton pump inhibitor used to decrease the amount of acid produced by the stomach. It is not a cough suppressant. NCSBN Client Need Topic: Physiological Integrity, Subtopic: Pharmacological therapies, Pediatric - Respiratory

The client using over-the-counter nasal decongestant drops reports unrelieved and worsening nasal congestion. What is the appropriate instruction for this client? A. Discontinue the medication for several days B. Use a combination of oral decongestant medications and drops for better results C. Switch to a stronger dose of the decongestant drops D. Increase the frequency of the nasal decongestant drops Submit Answer

Explanation Choice A is correct. Due to their local action, intranasal sympathomimetics produce few systemic effects. However, one side effect associated with their use is rebound congestion. Prolonged use causes hypersecretion of mucus and worsened nasal congestion once the drug effects wear off. This rebound effect sometimes leads to a cycle of increased drug use as the condition worsens. Because of the risk of rebound congestion, intranasal sympathomimetics should be used for no longer than 3-5 days. Prolonged use of decongestant drops (3 to 5 days) can lead to rebound congestion, which is relieved by discontinuing the medication for 2 to 3 weeks. Nasal congestion results from dilation of nasal blood vessels due to infection, inflammation, or allergy. With this dilation, there is a transudation of fluid into the tissue spaces, resulting in swelling of the nasal cavity. Nasal decongestants (sympathomimetic amines) stimulate the alpha-adrenergic receptors, producing vascular constriction (vasoconstriction) of the capillaries within the nasal mucosa. The result is a shrinking of the nasal mucous membranes and a reduction in fluid secretion (runny nose). Decongestants can make a client jittery, nervous, or restless. These side effects decrease or disappear as the body adjusts to the drug. When nasal decongestants are used for longer than 5 days, instead of the nasal membranes constricting, vasodilation occurs, causing an increased stuffy nose and nasal congestion. The nurse should emphasize the importance of limiting the use of nasal sprays and drops. As with any alpha-adrenergic drug (for example, decongestants), blood pressure and blood glucose levels can increase. These drugs are contraindicated and should only be used with extreme caution for clients with hypertension, cardiac disease, hyperthyroidism, and diabetes mellitus. Choice B is incorrect. The nurse should instruct the patient to discontinue the nose drops. Additionally, the nurse should not tell a client which medications to take. Instead, this responsibility belongs to the doctor. Choices C and D are incorrect. A more potent decongestant is not needed. Instead, the patient should stop using the drops (Choice C). The frequency should not be increased (Choice D). NCSBN Client Need, Topic: Physiological Integrity, Subtopic: Pharmacological & Parenteral Therapies, Allergies Last Updated - 07, Nov 2021

A patient with a chest tube drainage system has just been admitted to the unit. The nurse notes that the fluid in the water seal column is not fluctuating. The nurse knows that the best explanation of fluctuation cessation is that: A. There may be fibrin clots in the tubing B. The lung is collapsing C. There has been an increase in intrapleural pressure D. The tubing may have become dislodged from the chest Submit Answer

Explanation Choice A is correct. Fibrin clots from the lungs sometimes become lodged in the chest tube system resulting in the cessation of fluctuations in the water seal column. This may also occur when the lung becomes fully expanded. Choice B is incorrect. A collapsing lung is usually the indication for a chest tube. Choice C is incorrect. An increase in pleural pressure is healthy and fluctuates, causing movement in the water seal chamber. Choice D is incorrect. All functions and displays will cease to function correctly if the tubing is dislodged from the chest. This is not the most likely option. NCSBN client need Topic: Physiological Integrity, Reduction of Risk Potential

The nurse is taking care of a patient that was recently rescued from a near-drowning experience. The patient is now having pulmonary edema. The nurse understands that pulmonary edema is the result of which process? A. Water washing out the alveolar surfactant. B. Water introducing bacteria into the lungs and causing infection. C. Decreased intrathoracic pressure in the lungs. D. A sudden change in temperature within the lungs.

Explanation Choice A is correct. Freshwater and saltwater wash out the alveolar surfactant when they enter the lungs. This leads to alveolar collapse, intrapulmonary shunting, decreased lung compliance, and hypoxemia, which will eventually result in pulmonary edema. Choice B is incorrect. The introduction of bacteria into the lungs leading to infection may be possible; however, the initial result would be pneumonia, not pulmonary edema. Choice C is incorrect. A decreased intrathoracic pressure does not cause fluid to shift into the lungs. Choice D is incorrect. Sudden temperature changes may bring about cardiac dysrhythmias but is unlikely to cause pulmonary edema. Last Updated - 18, Jan 2022

The primary healthcare provider (PHCP) prescribes the client's chest tube discontinuation. The nurse should place which supply item at the bedside for this procedure? A. Suture removal kit B. Bag valve mask (BVM) C. Nasal cannula oxygen D. Wall suction with tubing

Explanation Choice A is correct. If the PHCP prescribes a chest tube to be discontinued, nursing should have pertinent supplies such as a suture removal kit, occlusive gauze, dry sterile gauze, tape, biohazard bag and a clamp. A suture removal kit is necessary because the chest tube is sutured into place. Choices B, C, and D are incorrect. A BVM is not pertinent in the discontinuation of a chest tube. This device is always at the bedside and necessary for a client receiving mechanical ventilation. Nasal cannula oxygen is not prescribed during or after the removal of a chest tube. If the chest tube is to be removed, this indicates improvement in their condition. If the client should experience complications following the removal of a chest tube, high flow oxygen will likely be utilized - not nasal cannula oxygen. Wall suction and tubing are not necessary to remove a chest tube. This may be necessary to operate the chest tube - but not for its removal. Additional Info The PHCP performs chest tube discontinuation. Nursing should have the pertinent supplies at the bedside (suture removal kit, occlusive gauze, dry sterile gauze, tape, biohazard bag and a clamp). This is a painful procedure so medicating the client with an opioid prior to this procedure is recommended. The sutures will be discontinued, the tape will be torn and prepared, as well as the gauze. The tube will be clamped, and while the client performs the Valsalva maneuver, the tube will be pulled and discontinued. The gauze will be applied and taped firmly. The device should be secured in a biohazard bag and disposed of accordingly. A post-procedure chest radiograph may be prescribed to determine if the initial problem has recurred. Last Updated - 26, Apr 2022

What is the priority nursing assessment for a 76-year-old patient with pneumonia? A. Airway patency B. Percussion sounds C. Breath sounds D. Respiratory rate Submit Answer

Explanation Choice A is correct. Impaired mobility in older adults creates a risk for airway collapse, reduced air exchange, hypoxia, hypercapnia, and acidosis. Reduced gag and cough reflexes can place older people at risk for aspiration of secretions and, potentially, aspiration pneumonia. There is a possibility of postoperative respiratory complications because of impaired cough reflex, weaker muscles, and decreased inspiratory capacity. Older adults are at increased risk of respiratory complications due to stress. The nurse should pay attention to maintaining adequate ventilation, keeping lung volumes high, clearing secretions, and positioning to prevent aspiration. Choices B, C, and D are incorrect. The airway always assumes priority in an assessment. NCSBN Client Need Topic: Physiological Integrity, Subtopic: Physiological Adaptation; Thorax and Lung Assessment Last Updated - 04, Feb 2022

The nurse is taking vital signs for a client who has a chest tube in place. While counting the client's respirations, the nurse notes that the water in the water-seal-chamber is fluctuating. Which action by the nurse is most appropriate based on this finding? A. Finish counting the client's respirations B. Empty the water-seal chamber C. Assist the client with incentive spirometry D. Notify the charge nurse

Explanation Choice A is correct. It is appropriate for the nurse to finish counting the client's respirations and continue to monitor them as normal. Fluctuations of water in the water-seal chamber with inspiration and expiration are a sign that the drainage system is patent. Normally, the water level will increase when the client breathes in, and then decrease when they breathe out. This is due to changes in intrathoracic pressures. Choice B is incorrect. The nurse should not empty the water-seal chamber. This would cause a break in the closed drainage system and could result in injury to the client. Choice C is incorrect. It is not necessary for the nurse to help the client with incentive spirometry (IS) based on this finding. The nurse has noted an expected finding of the chest-tube system and can continue to assess the client as normal. Choice D is incorrect. It is not necessary for the nurse to notify the charge nurse based on this finding. The nurse has noted an expected finding of the chest-tube system and can continue to monitor the client as normal. NCSBN Client Need Topic: Reduction of Risk Potential, Subtopic: Potential for Complications of Diagnostic Tests/Treatments/Procedures; Respiratory

The nurse is completing a home assessment for a client receiving oxygen therapy. Which essential piece of equipment should be available? A. Smoke detector B. Extension tubing C. Slip-resistant rugs D. Air humidifier Submit Answer

Explanation Choice A is correct. It is essential that a client receiving oxygen therapy have smoke detectors and a fire extinguisher as oxygen therapy enhances combustion. Having functional smoke detectors and a fire extinguisher is a priority because it promotes client safety. Choices B, C, and D are incorrect. Extension tubing may be prescribed to a client with oxygen therapy to assist them with activities of daily living. While this is quite helpful, it does not ensure client safety. Slip-resistant rugs are helpful in the reduction of falls, but this safety recommendation is not relevant to oxygen therapy. The oxygen source should be humidified if humidification is needed, not the room. Additional Info For a client being discharged with oxygen therapy, important teaching points to emphasize include: Have a pulse oximetry device readily available. Avoiding any open flame or heat. This includes an oven, stovetop, candles, matches, and cigarettes. Flammable products such as alcohol and oil should be avoided. Have working smoke detectors in the home as well as fire extinguishers. A sign posted on the door should be visible to alert visitors of the oxygen and extinguish any open flames before entry. Use a water-soluble jelly to lubricate the nasal passages and mouth to prevent drying. Last Updated - 28, Apr 2022

The nurse is caring for a 10-year-old client with a tracheostomy tube. The nurse notices that the client has a large amount of secretions and prepares the client for suction. Which action should the nurse take first? A. Hyperoxygenate the client B. Ask the client to take a deep breath C. Place the client in the supine position D. Notify the charge nurse Submit Answer

Explanation Choice A is correct. It is necessary to hyper-oxygenate the client prior to taking any of the other actions. This is one of the first steps in suctioning a tracheostomy. The nurse hyper-oxygenates the client to prepare them for the procedure and prevent oxygen desaturation. The nurse then inserts the suction catheter without suctioning to the pre-measured depth, applies intermittent suction, and rotates the suction catheter while removing it from the tracheostomy. Choice B is incorrect. It is not necessary to ask the client to take a deep breath prior to suctioning their tracheostomy. The nurse should hyper-oxygenate the client first. Choice C is incorrect. Placing the client supine is not specifically necessary for suctioning a tracheostomy. The client can be in any position of comfort where the tracheostomy can easily be accessed. For some clients, they may be sitting up in bed or up to the chair. For pediatric clients, it can be helpful to place a roll under their shoulders, hyperextending the neck so that the nurse can better reach the tracheostomy. Choice D is incorrect. It is not necessary to notify the charge nurse if the client needs suctioning. The nurse may proceed with this intervention. NCSBN Client Need Topic: Reduction of Risk Potential, Subtopic: Potential for Complications of Diagnostic Test/Treatments/Procedures; Respiratory Last Updated - 25, Jan 2022

When assessing a client's nose, the normal expected findings should be documented as: A. Nose symmetrical and midline B. Nose symmetrical with yellow drainage C. Nose asymmetrical with clear drainage D. Nose asymmetrical and proportional to facial features Submit Answer

Explanation Choice A is correct. Normal documentation of the assessment of the nose would include findings such as symmetrical, midline, without drainage, and proportional to facial features. Choice B is incorrect. Yellow nasal drainage is never a normal finding. Choices C and D are incorrect. The nose should be symmetrical. Last Updated - 11, Feb 2022

The nurse is working with a client who suffered a blunt injury to the chest wall. Which of the following assessment findings would indicate the presence of a pneumothorax? A. Diminished breath sounds B. A barrel chest C. Lower than normal respiratory rate D. A sucking noise at the site of the injury Submit Answer

Explanation Choice A is correct. Since this is a closed chest injury, the most common sign of pneumothorax (PTX) will be diminished breath sounds. Choice B is incorrect. A barrel chest occurs over time and indicates chronic obstructive pulmonary disease (COPD). Choice C is incorrect. With most cases of pneumothorax, the patient will become tachypneic rather than have a lower than usual respiratory rate. Choice D is incorrect. A sucking noise is noted in an open chest injury. NCSBN client need Topic: Physiological Integrity, Reduction of Risk Potential Last Updated - 24, Jan 2022

A 32-year-old man comes into the emergency department after being hit by a baseball bat in his chest. The nurse would suspect a pneumothorax because of which sign? A. Decreased respiratory rate B. Diminished breath sounds C. Presence of a barrel chest D. A sucking sound at the injury site Submit Answer

Explanation Choice B is correct. A client who experiences a pneumothorax may initially experience shortness of breath and chest pain. When the pneumothorax increases in size the client will display an increased respiratory rate, cyanosis, diminished breath sounds, and subcutaneous emphysema. Choice A is incorrect. The client who has a pneumothorax would display tachypnea (an increase in respiratory rate), not a decrease in respiratory rate. Choice C is incorrect. A barrel chest would indicate emphysema, a form of COPD. Patients with pneumothorax do not exhibit a barrel chest. Choice D is incorrect. The client's injuries are from a blunt object; therefore, the resulting pneumothorax would be a closed one. A sucking sound at the site of injury would denote an open chest injury. Last Updated - 06, Feb 2022

The nurse is assessing a client with chronic obstructive pulmonary disease (COPD). Based on the vital signs, what action should the nurse take? See the image below. A. Administer acetaminophen (APAP) B. Provide the patient with warm blankets C. Apply oxygen at 2 liters via nasal cannula D. Obtain an arterial blood gas (ABG) Submit Answer

Explanation Choice A is correct. The client has a fever, and the treatment for a fever includes fluids and antipyretics such as acetaminophen or ibuprofen. Choices B, C, and D are incorrect. The client's temperature is increased, and providing warm blankets would be detrimental, considering the patient has pyrexia. Tepid water baths may be used to promote comfort. Oxygen is not indicated as the oxygen saturation is above 88%, which is the norm for individuals with COPD. There is no indication in the question that the patient requires an ABG. The patient has tachypnea and hypoxia, a cardinal manifestation associated with COPD. Additional Info COPD is a combination of chronic bronchitis and pulmonary emphysema. Manifestations associated with COPD include tachypnea, hypoxia, hyperinflation of the lungs, and respiratory acidosis.

The nurse is reviewing a client's arterial blood gas (ABG) results which reveal the following: pH = 7.38; PaO2 = 72 mm Hg; PaCO2 = 39 mm Hg; HCO3- 24 mEq/L. Based on these results, the nurse should take which action? A. Administer supplemental oxygen B. Instruct the client to take deep breaths C. Administer sodium bicarbonate intravenously D. Reassess the ABG in two hours

Explanation Choice A is correct. The client's ABG indicates hypoxia. The normal PaO2 is 80-100; therefore, the nurse should give oxygen to address the situation. Choices B, C, and D are incorrect. Hypercapnia or a PaCO2 level higher than 45 mmHg indicates taking deep breaths to expel the excess carbon dioxide. The client's PaCO2 is 39, which does not necessitate deep breathing. Intravenous sodium bicarbonate should be administered when the client's HCO3 level is less than 22. Reassessing the ABG in two hours would not be appropriate because this client is hypoxic. Additional Info The normal PaO2 is 80-100. Values less than this need to be addressed promptly. Supplemental oxygen would be necessary to restore the values within normal limits. Last Updated - 16, Apr 2022

What action does the nurse perform to follow safe technique when using a portable oxygen cylinder? A. Check the amount of oxygen in the cylinder before using it. B. Use a cylinder for a patient transfer that indicates available oxygen is at 500 psi. C. Place the oxygen cylinder on the stretcher next to the patient. D. Discontinue oxygen flow by turning the cylinder key counter-clockwise until it is tight. Submit Answer

Explanation Choice A is correct. The cylinder must always be checked before use to ensure that enough oxygen is available for the patient. Choice B is incorrect. It is unsafe to use a cylinder that reads 500 psi or less because not enough oxygen remains for a patient transfer. Choice C is incorrect. A cylinder that is not secured correctly may result in injury to the patient during transfer. Choice D is incorrect. Oxygen flow is discontinued by turning the valve clockwise until it is tight. NCSBN Client Need Topic: Safe and Effective Care Environment, Subtopic: Safety and Infection Control; Providing Supplemental Oxygen Last Updated - 05, Feb 2022

The nurse is caring for a client with pneumonia receiving six liters a minute of nasal cannula oxygen. The client has a SpO2 of 81%, and the arterial blood gas (ABG) returns with a PaO2 of 68 mm Hg. Which immediate intervention should the nurse take? A. Notify the rapid response team (RRT). B. Obtain a prescription for a chest radiograph. C. Increase nasal cannula oxygen to seven liters a minute. D. Auscultate the lung fields for adventitious sounds. Submit Answer

Explanation Choice A is correct. This client demonstrates signs of acute respiratory distress syndrome (ARDS), a complication of pneumonia (hypoxemia). The client's inability to oxygen is highly concerning and is a classic manifestation of ARDS. An RRT should be immediately called to assist with appropriate interventions, including intubation by a qualified provider. Choices B, C, and D are incorrect. Assessment is not the priority because the provided information on the ABG and the SpO2 is worrisome and requires immediate intervention. A chest radiograph may be obtained to determine the presence of ARDS; however, this is not an initial intervention nor auscultating the lung fields. It would be inappropriate for the nurse to increase the nasal cannula oxygen to higher than six liters a minute because six is the maximum for this delivery device. The nurse should immediately consider applying a non-rebreather which will considerably maximize the amount of oxygen delivered to the client (it can deliver a FiO2 greater than 90%). Additional Info Inflammation from pneumonia may cause a client to develop ARDS. A classic manifestation of ARDS is hypoxemia (PaO2 less than 80 mm Hg). Adventitious lung sounds are not normally auscultated initially with ARDS and are not a reliable assessment. Treatment for ARDS is correcting the underlying cause and maintaining adequate oxygenation and ventilation via invasive and non-invasive means. The normal PaO2 is 80-100 mm Hg, and the normal SpO2 is greater than 95%. Last Updated - 27, Apr 2022

You are assessing a 35-year-old male patient in the clinic. He had a cough and intermittent abdominal pain for a few days. You receive results of an arterial blood gas that show: pH = 7.41 PaCO2 = 40 Bicarbonate = 25 You determine that this ABG shows: A. Normal ABG B. Respiratory acidosis C. Respiratory alkalosis D. Metabolic acidosis

Explanation Choice A is correct. This is a normal ABG. The registered nurse must know the basics of ABG interpretation, including the normal ranges for each of the values. First, the nurse should look at the pH. The normal range is 7.35-7.45. A value below 7.35 indicates an acidosis; a value above 7.45 indicates an alkalosis. The normal partial pressure of carbon dioxide (PaCO2) is 35-45 mmHg. Standard bicarbonate for a man this age is 22-29 mmol/L. Since this patient's values are all within the normal range, this is a normal ABG. The pH and PaCO2 define respiratory disorders. Respiratory acidosis is defined as a pH below 7.35 and a PaCO2 above 45 mmHg. Respiratory alkalosis is defined as a pH above 7.45 and a PaCO2 below 35 mmHg. Metabolic disorders are defined by the pH and the bicarbonate (HCO3). Metabolic acidosis is defined as a pH below 7.35 and an HCO3 below 22 mmol/L. Metabolic alkalosis is defined as a pH above 7.45 and an HCO3 above 29 mmol/L. Choices B, C, and D are incorrect. NCSBN Client Need Topic: Physiological Adaptation, Sub-Topic: Fluid and Electrolyte Imbalances, Respiratory Last Updated - 14, Feb 2022

Which of the following photos shows clubbing? A . B . C . D . Submit Answer

Explanation Choice A is correct. This is a photo showing the clubbing of the fingertips. Clubbing is defined as a bulbous enlargement of the ends of the fingers or toes. It is a sign of chronic hypoxia. Patients who experience hypoxia over long periods of time often experience clubbing in their fingertips due to the lack of oxygen reaching their distal extremities over time. Choice B is incorrect. This photo demonstrates fingertips that are cyanotic. Cyanosis is also a sign of hypoxia as the tissue is lacking sufficient oxygen and is not well perfused. Choice C is incorrect. This photo demonstrates edema or excessive swelling due to injury or inflammation. Edema can be a sign of many different physiological processes but typically represents a fluid volume excess. Choice D is incorrect. This photo demonstrates erythema or redness. Erythema can be a sign of many different things and can vary in severity. In general, it is due to irritation or inflammation. NCSBN Client Need: Topic: Psychosocial Integrity; Subtopic: Cardiac/Respiratory Last Updated - 08, Jan 2022

A nurse is preparing a client for a thoracentesis. All of the following are appropriate actions by the nurse except: A. Ensure the client has provided informed consent. B. Place the client in semi-Fowler's position. C. Instruct the client to remain still when the needle is inserted. D. Monitor for tachypnea, dyspnea, and cyanosis. Submit Answer

Explanation Choice B is correct. A thoracentesis is best performed with the client sitting upright and leaning slightly forward with arms supported. Unless there is a large volume of fluid in the pleural space, thoracentesis usually takes 10 to 15 minutes. During this time, most clients sit quietly on the edge of a chair or bed with their head and arms resting on a pillow positioned on a bedside table. Semi-Fowler's position is not utilized. Choice A is incorrect. The nurse should verify that the client has signed the informed consent form. If the client has not or the client has questions regarding the thoracentesis before signing the informed consent, the nurse should arrange for the physician to come to speak to the client. Choice C is incorrect. Typically, a local anesthetic (such as 1% lidocaine) is utilized to numb the area. Although numb, the insertion of the thoracentesis needle may be painful. Any sudden movement by the client could potentially force the hand through the pleural space, resulting in injuries to the visceral pleura and/or lung parenchyma. Therefore, the client should be instructed to remain still throughout the procedure. Choice D is incorrect. Any client undergoing a thoracic procedure should be monitored for tachypnea, dyspnea, and cyanosis (at minimum). Although complications are an overall rare occurrence in thoracentesis clients, when complications do occur, one of the more common complications is a pneumothorax. Learning Objective Identify the correct client positioning for a thoracentesis (i.e., the procedure requires the client to sit on the edge of a chair or bed with their head and arms resting on a pillow positioned on a bedside table). Additional Info Informed consent is part of the health care provider-patient relationship. Informed consent must be obtained and witnessed when the client is not under the influence of medication such as opioids. The person responsible for performing the procedure is responsible for obtaining informed consent. It has been standard practice at many facilities to obtain a chest x-ray after thoracentesis to rule out pneumothorax, document the extent of fluid removal, and view lung fields previously obscured by fluid. Source : Archer ReviewSource : Archer Review

The nurse is preparing a client for a total laryngectomy. When developing a plan of care for this client, the nurse recommends a consultation from which healthcare provider (HCP)? A. Endocrinology B. Respiratory therapy C. Dermatology D. Infectious disease Submit Answer

Explanation Choice B is correct. A total laryngectomy is the removal of the larynx and surrounding lymph nodes. This is a significant procedure that requires the placement of a tracheostomy. The client will need an interdisciplinary approach to their care. A central figure for a client with a tracheostomy is a respiratory therapist collaborating with the nurse regarding tracheostomy management. Choices A, C, and D are incorrect. Healthcare professionals essential in managing client care following a total laryngectomy include respiratory and speech therapy. Psychiatry may be useful later in the rehabilitation process because the client may develop a disturbed body image. Endocrinology, dermatology, and infectious disease are not relevant to the care of a client who underwent a total laryngectomy. Last Updated - 12, Sep 2022

The nurse has just finished assisting the surgeon with inserting a chest tube in a client with a pneumothorax. Which assessment finding indicates that the procedure has produced its desired effect? A. Consolidation is seen in the chest x-ray. B. Bilateral breath sounds are heard upon auscultation. C. There is rapid bubbling in the suction chamber of the chest drainage system. D. There is crepitus at the insertion site.

Explanation Choice B is correct. Bilateral breath sounds indicate that both the clients' lungs have expanded, which is the objective of the procedure. Choice A is incorrect. There is consolidation when fluid or exudates are present in the lungs, which indicates pneumonia. This shows deterioration in the status of the client. Choice C is incorrect. Rapid bubbling in the suction chamber indicates a leak. This is not an indication that the treatment is effective. Choice D is incorrect. Crepitus is a sign of subcutaneous emphysema that indicates an escape of oxygen into the surrounding tissues. This is not an indicator of lung expansion.

The nurse is performing a respiratory assessment of a patient with abnormal breathing patterns. The patient's breathing is rhythmic, yet has periods of apnea. The nurse would be most correct in charting this breathing style as: A. Neurogenic hyperventilation B. Cheyne-Stokes C. Apneustic D. Ataxic

Explanation Choice B is correct. Cheyne-Stokes is rhythmic breathing with periods of apnea usually caused by a metabolic issue or neurological problem. Choice A is incorrect. Neurogenic hyperventilation is regular and fast and indicates an issue in the pons or midbrain. Choice C is incorrect. Apneustic breathing is irregular and presents with apnea at the end and beginning of each breath. Choice D is incorrect. Ataxic breathing is utterly irregular in rate, depth, and rhythm.

Which of the following complaints should be first evaluated in a client with respiratory symptoms who has a history of asthma? A. An oxygen saturation of 94% B. Increased wheezing C. Sustained rhonchi D. Decreased respiratory rate Submit Answer

Explanation Choice B is correct. Clients with a history of asthma may be normal at the baseline. However, they exhibit symptoms and signs such as coughing, dyspnea, chest tightness, anxiety, tachypnea, and wheezing if having an asthma exacerbation. Wheezing is associated with airway inflammation and narrowing ( bronchospasm) that accompany asthma. Wheezes are high-pitched, continuous musical sounds that can be heard during inspiration and/ or expiration. Acute onset of wheezing may indicate an acute exacerbation in an asthmatic client. Bronchospasm should be treated, and often, a rescue inhaler with a short-acting bronchodilator is quickly administered to relieve symptoms. Choice A is incorrect. An oxygen saturation ( Sao2) reading at 94% is not a priority. Instead, pulse oximetry with Sao2 of less than 92% is a cause for concern. Choice C is incorrect. Rhonchi are low-pitched, continuous musical sounds heard during inspiration and/ or expiration. Bronchial rhonchi indicate secretions in the airway, such as those caused by pneumonia. Often, rhonchi are not associated with asthma. Choice D is incorrect. Should the client have an asthma exacerbation, increased respiration ( tachypnea) is expected, not decreased. NCSBN Client Need Topic: Physiological Integrity, Subtopic: Physiological Adaptation; Thorax and Lung Assessment Last Updated - 21, Jan 2022

A client is hospitalized for acute exacerbation of his COPD. The nurse taking care of him would expect to find which assessment finding? A. ABG showing a carbon dioxide level of 31 mmHg. B. An overinflated chest on chest x-ray. C. Improving oxygen saturation upon exercise. D. A wide diaphragm on chest x-ray. Submit Answer

Explanation Choice B is correct. In clients with COPD, there is a loss of elasticity in the lungs leading to congestion and hyperelasticity of the lungs, as seen on chest x-ray. Choice A is incorrect. In clients with COPD, carbon dioxide is trapped in the lungs resulting in an increased carbon dioxide level. Choice C is incorrect. Clients with COPD display a decrease in oxygen saturation during exercise due to airflow limitation. Choice D is incorrect. Clients with COPD display a flattened diaphragm, not a full diaphragm.

Which of the following would the nurse expect to be administered to treat a newborn with Respiratory Distress Syndrome (RDS) ? A. Theophylline B. Colfosceril C. Dexamethasone D. Albuterol Submit Answer

Explanation Choice B is correct. Colfosceril palmitate is a medication used as a pulmonary surfactant to treat and prevent respiratory distress syndrome (RDS). A fetus's lungs start making surfactants during the third trimester of pregnancy, or around 26 weeks gestation through labor and delivery. Surfactant coats the insides of the alveoli reducing the surface tension of fluid in the lungs, which helps make the alveoli more stable. This keeps the lungs from collapsing when the newborn exhales. Respiratory distress syndrome (RDS) is a type of neonatal respiratory disease that is most often caused by a lack of surfactant in the lungs. Prevention of RDS is generally desired in babies born at a gestational age less than 32 weeks. In an infant with RDS, colfosceril palmitate may be given via endotracheal tube in two to four doses during the first 24-48 hours after birth. Research shows that these surfactant medications improve respiratory status and decrease the incidence of pneumothorax. Choice A is incorrect. Theophylline is used to treat the symptoms of asthma or other lung conditions that block the airways, such as emphysema or chronic bronchitis. Choice C is incorrect. Corticosteroids are typically given to pregnant women below 34 weeks gestation who are at high risk for preterm delivery. Corticosteroids promote the development of type 1 and type 2 pneumocytes. As the infant approaches "term" gestation, the surfactant is secreted by type 2 pneumocytes. Given antenatally to those women at risk of pre-term delivery, dexamethasone reduces the incidence and severity of RDS in the infants. Postnatally, surfactant administration is the mainstay of treating RDS. Using steroids as frontline treatment in RDS may have growth and neurodevelopmental adverse effects. However, dexamethasone may be considered in infants who require mechanical ventilation between 7 and 21 days of age, who are receiving supplemental oxygen, and are at high risk of neonatal chronic lung disease (CLD). Choice D is incorrect. Albuterol is a bronchodilator taken by mouth to treat bronchospasm. It is for use in adults and children at least six years old. NCSBN Client Need - Topic: Physiological Integrity; Subtopic: Pharmacological Therapies

The nurse is caring for assigned clients. The nurse should immediately follow up with the client who A. has influenza and their most recent temperature was 102°F (39°C). B. is recovering from a thoracentesis and reports a nagging cough. C. reports reddish-brown sputum immediately following a bronchoscopy. D. has pulmonary tuberculosis and is wearing a surgical mask while ambulating to radiology. Submit Answer

Explanation Choice B is correct. Following a thoracentesis, the nurse must assess the client for the most common complication of pneumothorax. Manifestations of a pneumothorax that are concerning include a nagging persistent cough, increased heart and respiratory rate, dyspnea, and potentially a feeling of air hunger. The nurse must act quickly because the client's condition may deteriorate. Depending on the size of the pneumothorax, a chest tube may be needed. Choices A, C, and D are incorrect. A fever is common with influenza and would not necessitate the need for immediate follow-up. Following a bronchoscopy, reddish-brown sputum is expected because as the scope passes by the mucosa, it may irritate. Finally, no follow-up is necessary for a client with pulmonary tuberculosis wearing a surgical mask. This is an appropriate infection control measure. It is the healthcare worker that should wear the respirator (N95 mask). Additional Info Thoracentesis is the needle aspiration of pleural fluid or air from the pleural space for diagnostic or management purposes. This test can be performed at the bedside and typically involves using ultrasound to guide the needle. Nursing considerations for this procedure involve witnessing the informed consent, positioning the client over a bedside table, and supporting the client during the procedure. The provider will insert a needle (after the skin has been anesthetized) attached to a syringe and will slowly aspirate fluid. This fluid may be sent for laboratory analysis. A sterile pressure dressing will be applied, and a follow-up chest x-ray will be performed. The most common complication following this procedure is a pneumothorax.

A 70-year-old client was admitted for pneumonia. The client develops acute respiratory distress syndrome that resulted in respiratory arrest, requiring the need for an endotracheal tube. Attempts to wean him from mechanical ventilation were ineffective, and a tracheostomy was created. How can the nurse minimize bleeding around the insertion site for the first 24 hours following tracheostomy? A. Deflate the cuff for 10 minutes every other hour instead of 5 minutes every hour. B. Avoid manipulating the tracheostomy and do not deflate the cuff. C. Watch out for signs of crepitus and report this immediately to the physician. D. Change tracheostomy dressing PRN using 1/2 strength hydrogen peroxide to clean the site. Submit Answer

Explanation Choice B is correct. For the first 24 hours after tracheostomy insertion, the cuff should not be deflated. To minimize bleeding, it is best to avoid any manipulation, including cuff deflation. Choices A, C, and D are incorrect. Small amounts of crepitus are expected, but large quantities or increasing size of the area of crepitus should be reported. The tracheostomy dressing may be changed PRN, but site care should be done using a standard saline solution. Additional Info Source : Archer ReviewSource : Archer Review Last Updated - 20, Jan 2022

A client with a left-sided pneumothorax had a chest tube inserted 3 hours ago. There is no fluctuation in the water-seal chamber of the Pleurovac. What should be the nurse's first action? A. Auscultate the client's chest wall. B. Assess the tubing for any kinks. C. Instruct the client to take deep breaths. D. Ask the client to turn from side to side.

Explanation Choice B is correct. If there is no fluctuation in the water seal chamber, the nurse should first check the integrity of the chest tubes from the client's chest wall down to the Pleurovac for dependent loops or kinks. Choice A is incorrect. The nurse should assess for breath sounds but should determine why there is no fluctuation in the water-seal chamber first. Re-expansion of the lungs after 3 hours is too early to occur. Choice C is incorrect. The nurse should check the tubing for kinks or dependent loops first; afterward, the nurse can tell the client to deep breathe and cough to push out blockages through the pipe. Choice D is incorrect. Turning the client does not aid in troubleshooting the problem for the client.

The nurse in the ER is caring for an Asian-American with an acute asthma attack. When assessing the client, the nurse understands that which of the following information holds the least priority? A. History of present illness B. Psychosocial assessment C. Neurological status D. Vital signs and oxygen saturation Submit Answer

Explanation Choice B is correct. In Asian American culture, asking personal questions during the initial meeting is uncomfortable and indiscreet. The nurse can put off this assessment until the patient is already relaxed and comfortable. Choice A is incorrect. Assessing for the history of present illness is necessary, providing information that will help direct client care. Choice C is incorrect. Following the hierarchy of needs, the physiological concerns of the patient should be prioritized over other aspects. Neurological status should be assessed as soon as possible. Choice D is incorrect. Following the hierarchy of needs, the physiological concerns of the patient should be prioritized over other aspects. Oxygen saturation and vital signs should be assessed with a high priority. Last Updated - 15, Feb 2022

The nurse is developing a plan of care for a client admitted P. aeruginosa pneumonia. Which of the following should the nurse include in the client's plan of care? A. Instruct the client to wear an N95 mask when ambulating in the hall. B. Initiate a vascular access device and encourage by-mouth fluids. C. Obtain daily weights every morning using the same scale. D. Administer prescribed oseltamivir within 48 hours of symptom onset.

Explanation Choice B is correct. Initiating vascular access is essential for a client admitted with P. aeruginosa pneumonia because parenteral antibiotics are the mainstay of treatment. Dehydration is common in pneumonia, and encouraging non-caffeinated fluids is beneficial. Choices A, C, and D are incorrect. The client should wear a surgical mask as they ambulate around the nursing unit. Droplet precautions will be instituted, and the essential PPE for droplet precautions includes a surgical mask. Daily weights are not part of care standard of care for a client with pneumonia as fluid volume overload is unlikely. Oseltamivir is indicated for influenza infections, not bacterial pneumonia. NCLEX Category: Physiological adaptation Activity Statement: Illness management Question type: Application Additional Info For a client with P. aeruginosa pneumonia, the nurse should initiate droplet precautions and maintain airway patency. The biggest complication associated with pneumonia is acute respiratory distress syndrome or sepsis. Obtaining blood cultures, providing pulmonary hygiene, and initiating prescribed antibiotics are essential in managing pneumonia. Last Updated - 04, Dec 2022

The nurse is caring for a client with chronic obstructive pulmonary disease (COPD) who has an increased red blood cell count (RBC). The nurse understands that this finding is likely from A. Increased carbon dioxide levels B. Low blood oxygen levels C. Insensible water loss D. Decreased fluid intake Submit Answer

Explanation Choice B is correct. Low blood oxygen levels, which is a clinical feature associated with COPD, cause the kidneys to respond by releasing erythropoietin (EPO), which stimulates red blood cell production. The red blood cell count is elevated to compensate for hypoxia or low oxygen levels. More cells are available to carry and deliver the maximum amount of oxygen. Polycythemia is a condition with increased red blood cells in the blood. Choices A, C, and D are incorrect. Hypercapnia, which is too much carbon dioxide in the blood, would not contribute to an increase in the red blood cell count (RBC). While dehydration (caused by sensible or insensible fluid loss) would cause hemoconcentration, dehydration is not a clinical finding linked to COPD. Additional Info Source : Archer Review Polycythemia is a condition in which the number of RBCs in the blood is greater than normal. The blood hemoglobin levels in PV are sustained at greater than 18 g/dL in men or greater than 16.5 g/dL in women. When a client has COPD, they commonly have secondary polycythemia caused by the hypoxia which prompts erthropoesis.

Your newly assigned client has a history of chronic obstructive pulmonary disease (COPD). When you enter his room, you find his oxygen is running at 6 L/min, his color is flushed, and his respirations are 8/min. What should you do first? A. Place client in high Fowler's position B. Lower the oxygen rate C. Take baseline vital signs D. Obtain an EKG

Explanation Choice B is correct. Low oxygen level stimulates respiration. A high concentration of supplemental oxygen removes the hypoxic drive to breathe. This can lead to increased hypoventilation and possibly the development of or worsening of respiratory acidosis can occur. Left untreated, this can result in a patient's death. Individuals with COPD experience lowered oxygen tension and increased carbon dioxide retention during sleep, especially during REM sleep, when neuromuscular control usually is depressed. This can result in pulmonary spasms and transient pulmonary hypertension. Choice A is incorrect. Although High-Fowler's position is recommended, it is not the first action that should be taken. Choice C is incorrect. Baseline vitals are taken on admission to the unit. Choice D is incorrect. While an EKG may be ordered if symptoms do not resolve, the first nursing action should be to lower the O2 rate and see if there is an improvement in the patient's status. NCSBN Client Need Topic: Physiological Integrity, Subtopic: Reduction of Risk Potential

The nurse is caring for a client receiving mechanical ventilation. Which prescription from the primary healthcare physician (PHCP) should the nurse anticipate? A. Hydroxyzine B. Pantoprazole C. Rivastigmine D. Verapamil Submit Answer

Explanation Choice B is correct. Mechanical ventilation may cause a stress ulcer. A proton pump inhibitor (PPI) or a histamine-2 receptor antagonist (H2 blocker) may be utilized to prevent this ulcer which may lead to a gastrointestinal bleed. Choice A, C, and D are incorrect. Hydroxyzine is an anticholinergic utilized in allergic reactions and anxiety. Rivastigmine increases acetylcholine in the central nervous system and is indicated for dementia. Verapamil is a calcium channel blocker indicated for hypertension and migraine headache prophylaxis. These medications have no relevance to mechanical ventilation management. Additional Info Mechanical ventilation poses a risk for a stress ulcer to form. This is caused by hypersecretion of gastric acid and impaired protection of the gastric mucosa. Stress ulcers pose a serious risk as they may cause gastrointestinal bleeding. This bleeding may lead to perforation and then shock. PPIs such as pantoprazole or H2 blockers famotidine may be used to mitigate this risk. Manifestations of stress ulcers include hematemesis, melena, anemia, and shock. Last Updated - 26, Apr 2022

The client with chronic obstructive pulmonary disease (COPD) reports trouble sleeping at night. Which question is most important for the nurse to ask? A. "What do you eat before you go to bed? B. "How many pillows do you sleep on at night?" C. "Have you always been a light sleeper?" D. "Is your partner snoring and keeping you awake?" Submit Answer

Explanation Choice B is correct. Orthopnea is shortness of breath that occurs when lying flat, causing the person to have to sleep propped up in bed or sitting in a chair. Asking the client how many pillows they use to sleep on is a way to assess if the client has been educated about measures to prevent orthopnea. COPD causes blocked or narrowed airways that make breathing more difficult. Clients may experience symptoms like wheezing, coughing, mucus production, and tightness in the chest. Smoking or exposure to harmful chemicals can cause COPD. Orthopnea is a common symptom of COPD clients. Choices A, C, and D are incorrect. While some foods may aggravate reflux or create a feeling of being too full, which can disrupt sleep, this is not the most appropriate answer choice concerning shortness of breath with COPD. Being a light sleeper or having a partner who snores may interrupt sleep. However, the nurse's assessment should first address ways the client can make adjustments to prevent sleep disruption.

The nurse cares for a client receiving positive end-expiratory pressure (PEEP) while being mechanically ventilated. The nurse understands that this setting is used to A. give a set amount of inspiratory pressure. B. prevent closure of the small airways during expiration. C. allow spontaneous breaths between mandatory ones. D. deliver a preset tidal volume with each breath.

Explanation Choice B is correct. Positive end-expiratory pressure (PEEP) is used in clients with acute respiratory distress syndrome (ARDS) because it improves lung compliance and oxygenation. This is accomplished by adding pressure at exhalation to keep the alveoli open. PEEP is a setting that may be added to a variety of ventilator modes. Choices A, C, and D are incorrect. PEEP does not give an amount of pressure upon inspiration. This description is appropriate for pressure support ventilation which provides pressure when the client takes a spontaneous breath. A client allowed to take spontaneous breaths between mandatory ones is a SIMV and AC modes feature. The critical difference between SIMV and AC is that when a client takes a spontaneous breath, they are forced to take in the prescribed tidal volume, whereas SIMV they can take in their own tidal volume during that spontaneous breath. PEEP does not have anything to do with tidal volumes, and giving a preset tidal volume with each breath would be an appropriate description for the AC mode. Additional Info When caring for a client on a ventilator, you should be familiar with the following settings: Mode (Volume [SIMV, A/C] or Pressure [PSV]) Rate (Number of breaths per minute) Tidal volume (the amount of gas delivered to the client) Fraction of inspired oxygen (FiO2 - the percentage of oxygen given per breath) PEEP (pressure added at exhalation to keep the small airways open and mitigate atelectasis) Pressure support (PS - provides added pressure when the client takes a spontaneous breath)

The nurse is caring for a client in the emergency department. The client is short of breath upon arrival to the ED and is coughing up purulent sputum. Oxygen is being administered at 2 liters per minute via nasal cannula. The client's blood pressure is 100/58 mmHg, pulse is 88, and respiratory rate is 24. The client is afebrile with an oxygen saturation of 92%. The results of arterial blood gas testing are: pH = 7.25, PaO2 = 93, PaCO2 = 69, and HCO3 = 25. The nurse understands that this ABG shows: A. Respiratory alkalosis B. Respiratory acidosis C. Metabolic alkalosis D. Metabolic acidosis Submit Answer

Explanation Choice B is correct. Respiratory acidosis. The trick to interpreting ABGs is to know the normal values and to use a systematic process for interpretation. Normal values for ABGs are pH: 7.35-7.45, PaO2 = 75-100 mmHg, PaCO2 = 35-45 mmHg, HCO3 = 22-26 mEq/L, and O2 sat = 94-100%. First, look at the pH. In this case, the pH is < 7.35, which indicates an acidotic condition. Second, examine the PaCO2. In this case, the value is > 45 mmHg, which indicates this is respiratory acidosis. Choices A, C, and D are incorrect. NCSBN Client Need Topic: Reduction of Risk Potential, Sub-topic: Diagnostic Tests, Respiratory

The nurse is caring for a client following a bedside thoracentesis. Which action should the nurse take immediately following the procedure? A. Instruct the client to take slow, shallow breaths B. Assess the patient's respiratory status. C. Label the lab specimen for culture. D. Provide nasal cannula oxygen. Submit Answer

Explanation Choice B is correct. Respiratory complications following rapid removal of fluid include hypoxemia and pulmonary edema. Assessing the client's respiratory status is a high priority and would be the first action the nurse should take. Choices A, B, C, and D are incorrect. A dressing should be applied following the removal of the needle, and it would be an appropriate action for the nurse to monitor the site and the drainage. A scant amount of blood would be expected following the puncture, not necessarily signifying a complication. The client should be instructed to breathe deep after this procedure to assist with the reexpansion of the lung, but this would not prioritize over an immediate respiratory assessment. Sending specimens to the lab would be an appropriate action but should be done at the time of collection. Supplying nasal cannula oxygen is not necessary unless the respiratory assessment reveals that the client is in distress. Additional Info Following a thoracentesis, immediate assessments include the client's vital signs and listening to the lungs for absent or reduced sounds on the affected side (this could indicate a pneumothorax). Check the puncture site and dressing for leakage or bleeding. Assess for complications, such as reaccumulation of fluid in the pleural space, subcutaneous emphysema, infection, and tension pneumothorax. Urge the client to breathe deeply to promote lung expansion. Last Updated - 27, Apr 2022

The nurse is caring for a client with a pulmonary embolism (PE). Which of the following findings require immediate follow-up? A. Pleuritic chest pain B. Restlessness C. Cough D. Exertional dyspnea

Explanation Choice B is correct. Restlessness is an ominous sign suggestive of hypoxia. Hypoxia indicates pulmonary embolism (PE) that is advancing, and the patient is becoming unstable. The nurse should immediately follow up on this finding. Choices A, C, and D are incorrect. Pleuritic chest pain, cough, and exertional dyspnea are all expected features of a PE. Individuals with a PE may be asymptomatic depending on the size of the PE. Additional Info Pulmonary embolism (PE) risk factors include venous thromboembolism, hypercoagulability, immobility, and smoking. Most PEs arise from an untreated VTE that may be lethal if it goes unrecognized. Manifestations of a PE include pleuritic chest pain, cough, and dyspnea. Last Updated - 02, Feb 2022

A client with a history of smoking has an increased risk in the development of which of the following? A. Raynaud disease B. Peripheral arterial disease (PAD) C. Deep vein thrombosis (DVT) D. Venous insufficiency Submit Answer

Explanation Choice B is correct. Smoking is one of the most devastating risk factors associated with peripheral arterial disease. (PAD). Choice A is incorrect. Raynaud disease is characterized by spasms of the arteries in the extremities, especially the fingers (Raynaud's phenomenon). It is typically brought on by constant cold or vibration and leads to pallor, pain, numbness, and in severe cases, gangrene. Choice C is incorrect. Deep vein thrombosis (DVT) is the formation of a blood clot in a deep vein, most commonly in the legs or pelvis. Factors that increase the risk of developing DVT include: injury to a vein that is often caused by fractures, severe muscle injury, or major surgery (mainly involving the abdomen, pelvis, hip, or legs). Choice D is incorrect. In healthy veins, there is a continuous flow of blood from the limbs back toward the heart. Valves within the veins of the legs help prevent the backflow of blood. Either blood clots or varicose veins most often cause venous insufficiency. NCSBN Client Need Topic: Health Promotion and Maintenance, Subtopic: Peripheral Vascular Risk Factors Last Updated - 07, Feb 2022

The nurse is supervising a student. While discussing the assessment process, the nurse should educate the student that the best way to assess the client's respiratory rate is by: A. Place one hand over the patient's chest and count for 30 seconds. B. Count the respirations for one minute with your hand placed on the client's radial pulse. C. Observe and count respirations for 30 seconds, then multiply by two without mentioning to the client that the respirations are being counted. D. If respirations are irregular, ask the patient to rest for 10 minutes, then reassess the respiratory rate. Submit Answer

Explanation Choice B is correct. The best way to assess respiratory rate is to count the respirations for a full minute ( 60 seconds). One breath is equal to one rise ( inspiration) and one fall ( expiration) of the chest wall. While assessing respiration, the pattern ( regular/ irregular) and the effort ( labored/ unlabored) are also assessed in addition to the rate. A full minute length will provide accurate assessment of the respiratory rate and the pattern. If the clients know their respiration is being observed, they may alter their breathing. To prevent this, the nurse should attempt to count respirations simultaneously while checking the pulse rate or a pulse oximetry reading. Choice A, C, and D are incorrect. Placing a hand on the patient's chest is likely to cause him/her to change their rate/depth of respiration. Also, respirations should be counted for one full minute, not 30 seconds. A change in the respiratory rate gives critical information regarding a client's baseline status, distress, recovery or decline. If the respirations are irregular, the nurse should fully assess for one minute and subsequently, document the rate and the pattern. Postponing the respiratory assessment could mean ignoring the possible signs of distress. NCSBN Client Need Topic: Health Promotion and Maintenance; Subtopic: Respiration Last Updated - 13, Feb 2022

The nurse is assessing a client with a chest tube for crepitus. Which assessment technique is most appropriate for the nurse to perform? A. Press down on the client's abdomen, releasing, and assessing for pain. B. Palpate the skin around the chest tube and observe for a crackling sensation. C. Auscultating the bowel sounds in each quadrant. D. Inspect the client's chest for an even rise and fall.

Explanation Choice B is correct. The nurse may assess for crepitus by palpating the skin around the chest tube and observing for a crackling sensation. Crepitus is defined as infiltration of air in the subcutaneous layer of the skin, also known as subcutaneous emphysema. It is caused by air leaking into the subcutaneous space. Choice A is incorrect. When the nurse presses down on a client's abdomen and then releases it, the nurse is assessing for rebound tenderness. This occurs when pain is present upon letting go of the client's abdomen, not pressing inward; this is a sign of peritonitis. Choice C is incorrect. Auscultating the bowel sounds in each quadrant is not an appropriate way to assess for crepitus. Choice D is incorrect. Inspecting the client's chest for even rise and fall will not allow the nurse to monitor for crepitus; rather this will help the nurse to assess for a symmetrical chest and unlabored breathing. NCSBN Client Need: Topic: Reduction of Risk Potential, Subtopic: System Specific Assessments, Respiratory Last Updated - 15, Nov 2021

When instructing a post-surgical patient with an abdominal incision on deep breathing and coughing, the nurse explains that the patient should be sitting up for these activities because: A. It is physically more comfortable for the patient B. Helps the patient to support their incision with a pillow C. Loosens respiratory secretions D. Allows the patient to observe their area and relax

Explanation Choice B is correct. The patient should be sitting when deep breathing and coughing. This position allows the patient to support his incision with a pillow, providing abdominal support when coughing. It also allows the lungs to expand more fully because it enables the diaphragm to move downwards under gravity. Coughing and deep breathing exercises are essential to enhance lung expansion and mobilize secretions, thereby preventing atelectasis (collapse of the alveoli) and pneumonia. Instructions on deep breathing exercises should include: Place the palms down on the rib cage's border and inhale slowly and evenly through the nose until the enormous chest expansion is achieved. Hold the breath for 2 to 3 seconds. Then exhale slowly through the mouth. Continue exhalation until maximum chest contraction has been achieved Choice A is incorrect. The supine position is more comfortable. However, it does not permit the lungs to fully expand. Choice C and D are incorrect. There is no association between loosening respiratory secretions or relaxation when in the sitting position. The sitting position allows lung expansion and can help mobilize the respiratory secretions, not necessarily loosen the secretions. NCSBN Client Need Topic: Physiological Integrity; Subtopic: Basic Care and Comfort Last Updated - 04, Dec 2021

A client is diagnosed with a spontaneous pneumothorax which results in the need to insert a chest tube. What is the best explanation for the nurse to provide this client? A. "The tube will prevent you from having chest pains." B. "The tube will remove excess air from your chest." C. "The tube controls the amount of air that enters your chest." D. "The tube will seal the hole in your lung." Submit Answer

Explanation Choice B is correct. The purpose of the chest tube is to create negative pressure and remove the air that has accumulated in the pleural space. Choice A is incorrect. Chest tubes do not prevent chest pain. Many patients complain of pain and discomfort because of the tube. However, the necessity of removing air is paramount. Choice C is incorrect. The purpose of the chest tube is to remove air that has accumulated, not control the amount of air entering the lung. Choice D is incorrect. The chest tube does not seal a hole in the lung. NCSBN Client Need Topic: Physiological Integrity, Subtopic: Reduction of Risk Potential

Following a persistent cough, chills, and fever, a client was admitted for a possible respiratory infection. The admission orders include a regular diet, vital signs every 4 hours, ampicillin 250 mg PO every 6 hours, and sputum culture. Before beginning antibiotic therapy, the nurse should perform which of the following? A. Provide the client a full meal B. Collect the sputum sample C. Assess the client's vital signs D. Assess the client's oxygen saturation Submit Answer

Explanation Choice B is correct. When caring for a client requiring a sputum culture, the sputum sample should be obtained before initiating antibiotic therapy. Obtaining the sputum sample prior to initiating antibiotic therapy allows for accurate detection of the organism(s) causing the infection through the sputum culture. Choice A is incorrect. Ampicillin should be given on an empty stomach (at least 30 minutes before or two hours after a meal). Choice C is incorrect. Overall, the assessment of a client's vital signs are an essential nursing action. Conversely, although obtaining new vital signs for any client prior to administering medication is always a sound practice, there is no specific indication or need to do so before administering oral ampicillin. Choice D is incorrect. Overall, assessing a client's oxygen saturation is an essential nursing action. Conversely, although obtaining a new oxygen saturation for any client before administering medication is always a sound practice, there is no specific indication or need to do so before administering oral ampicillin. Learning Objective In a client with admission orders for both a sputum culture and antibiotic therapy, recognize the need to obtain the sputum culture prior to initiating the antibiotic therapy. Additional Info In this case, the client presented with a generalized respiratory illness. The admission orders included obtaining a sputum culture and initiating a powerful, broad-spectrum antibiotic (i.e., ampicillin). The sputum culture (and subsequent sensitivity) will be sent to the lab, where it will develop and reveal what specific organism is causing the infection. Once the organism has been identified, the health care provider (HCP) will switch the broad-spectrum antibiotic (i.e., ampicillin) to an antibiotic chosen to target the identified affecting organism. Conversely, if the ampicillin is initiated prior to the collection of the sputum culture, a decrease of the culture sensitivity occurs, resulting in a loss of data. This data loss jeopardizes the ability of the sputum culture tests to accurately pinpoint what organism to target, resulting in a decreased ability to accurately determine the most effective antibiotic(s). Last Updated - 18, Oct 2022

You are caring for a patient with blood clots in his lungs. He is receiving urokinase for treating pulmonary embolism. The urokinase has been infusing for the last 10 hours. As you assess the patient, you note that his blood pressure is 102/64, heart rate is 108, and his respiratory rate is 16 breaths per minute. The patient asks to use the bedpan. When he is finished, you notice that he has passed a medium-sized bloody stool. Your best intervention is to: A. Closely monitor the patient B. Stop the urokinase and call the physician C. Administer Vitamin K intramuscularly D. Slow the administration of urokinase

Explanation Choice B is correct. You should immediately stop the urokinase and call the physician. Urokinase is a thrombolytic medication used in the treatment of blood clots. It is given over 12 hours through an intravenous site. One of the severe side effects of urokinase is bleeding. The bleeding can be from any location, including internal bleeding in the abdomen that can result in bloody stools. Although the team will closely monitor the patient, the nurse should immediately stop the urokinase and call the physician for further orders. Choices A, C, and D are incorrect. Slowing the intravenous infusion of urokinase will not help to control the bleeding. The nurse should anticipate the administration of Amicar, which is used to treat bleeding associated with the use of fibrinolytic. Vitamin K will not do anything to reverse the fibrinolytic. NCSBN Client Need Topic: Pharmacological and Parenteral Therapies, Sub-Topic: Adverse Effects/Contraindications/Side Effects/Interactions, Respiratory Last Updated - 26, Oct 2021

You are caring for a 55-year-old male patient in the emergency department. He has a history of chronic obstructive pulmonary disease (COPD). He came to the ED with a complaint of shortness of breath. His respiratory rate is 28 per minute, and his breaths are shallow and somewhat difficult. You put him on supplemental oxygen at 2 L/minute. You draw ABGs. You receive results of the arterial blood gas that show: pH = 7.30 PaCO2 = 49 Bicarbonate = 25 You determine that this ABG shows: A. Metabolic alkalosis B. Respiratory acidosis C. Respiratory alkalosis D. Metabolic acidosis Submit Answer

Explanation Choice B is correct.This ABG shows a respiratory acidosis. The first clue in this patient is the diagnosis of COPD. In COPD, the patient suffers from severe hypoventilation. This hypoventilation results in the retention of carbon dioxide. The registered nurse must know the basics of ABG interpretation, including the normal ranges for each of the values. First, the nurse should look at the pH. The normal range is 7.35-7.45. A value below 7.35 indicates an acidosis; a value above 7.45 indicates an alkalosis. The normal partial pressure of carbon dioxide (PaCO2) is 35-45 mmHg. Standard bicarbonate for a man this age is 22-29 mmol/L. The pH in this patient shows that the condition is acidosis. The high PaCO2 indicates that it is a respiratory problem. These values would support the assumption based on the diagnosis of COPD. The pH and PaCO2 define respiratory disorders. Respiratory acidosis is defined as a pH below 7.35 and a PaCO2 above 45 mmHg. Respiratory alkalosis is defined as a pH above 7.45 and a PaCO2 below 35 mmHg. Metabolic disorders are defined by the pH and the bicarbonate (HCO3). Metabolic acidosis is defined as a pH below 7.35 and an HCO3 below 22 mmol/L. Metabolic alkalosis is defined as a pH above 7.45 and an HCO3 above 29 mmol/L. Choices A, C, and D are incorrect. NCSBN Client Need Topic: Physiological Adaptation, Sub-Topic: Fluid and Electrolyte Imbalances, Respiratory

The nurse is caring for a client with a chest tube for the treatment of a pneumothorax. Which item is essential to have at the bedside? A. Nasal cannula oxygen B. Tracheostomy set C. Bottle of sterile water D. An ampule of Dextrose 50% Submit Answer

Explanation Choice C is correct. A bottle of sterile water is essential to have at the bedside because if the chest tube becomes disconnected from the chest tube system, the nurse can maintain the patency of the system by putting the end of the tube in sterile water, which will prevent air from reentering the pleural space. Choices A, B, and D are incorrect. Nasal cannula oxygen is unnecessary because if the chest tube becomes disconnected, the highest priority is to fix the underlying problem (chest tube disconnection). A tracheostomy set is not necessary for a client with a chest tube. Rather, this is something for a client acutely recovering from a thyroidectomy. An ampule of Dextrose 50% is useful for severe hypoglycemia and has no utility in managing an emergency with a chest tube. Additional Info A chest tube is a primary treatment for a client with a pneumothorax. The nurse is responsible for assessing the patency of the system by assessing each chamber. The nurse should always ensure client safety by having the appropriate emergency equipment at the bedside. This equipment includes occlusive sterile gauze, a bottle of sterile water, and clamps.

A client is scheduled to undergo an outpatient diagnostic bronchoscopy. The client's spouse transported the client to the outpatient clinic and will provide all care following the client's discharge. Out of the following nursing diagnoses, which is the most appropriate for this client? A. Risk for impaired skin integrity related to immobilization B. Risk for infection related to an invasive procedure C. Risk for bleeding related to diagnostic bronchoscopy D. Lack of knowledge regarding postoperative care related to inexperience with diagnostic bronchoscopy as evidenced by frequent queries about the postoperative routine Submit Answer

Explanation Choice C is correct. A diagnostic bronchoscopy is an invasive procedure that allows the direct examination of the larynx, trachea, and bronchi using a bronchoscope to obtain tissue samples to obtain a biopsy and/or culture, often to assist with cancer staging or assist with the removal of secretions unable to be cleared with standard suctioning procedures. The most common complication of a diagnostic bronchoscopy is bleeding, as samples are taken during the procedure as necessary. Choice A is incorrect. On average, a diagnostic bronchoscopy, during which time the client will be placed in a sitting or supine position, lasts approximately 30 to 60 minutes. Much of the remaining time is spent monitoring the patient in recovery. Therefore, the nursing diagnosis "[r]isk for impaired skin integrity related to immobilization" is incorrect, as the client will not be in an immobilized position for a duration significant enough to produce notable risk for impaired skin integrity. Choice B is incorrect. With each break in the skin, the risk for infection increases. While this risk is minimized in both outpatient and inpatient environments by utilizing best practices, the risk remains present, as the procedure is an invasive procedure that, although rare, could favor infectious complications. Conversely, the risk for infection is so low for a diagnostic bronchoscopy, this is not the correct answer. Choice D is incorrect. Here, since the spouse will be the individual providing the majority of the client's post-procedure care, the client's lack of knowledge regarding postoperative care is not as alarming as it would be if the client's spouse displayed a lack of knowledge or understanding. Here, the nurse should continue to educate the client before administering any form of sedation, utilizing verbal and written materials while the spouse is also in the room. While both the client and the client's spouse should be provided post-procedure instructions, the priority is that the client's spouse demonstrates comprehension of these discharge instructions to staff. Learning Objective Identify and prioritize the risk for bleeding as the most appropriate nursing diagnosis for a client scheduled to undergo an outpatient diagnostic bronchoscopy. Additional Info During a diagnostic bronchoscopy, a bronchoscope is passed through the client's nose or mouth. Changes in breathing patterns, chest pain, and/or oxygen saturation levels should be monitored, documented, and reported. The client should be closely monitored until the effects of the sedation have resolved and the client's gag reflex has returned. Following the procedure, the client should continue to be monitored for an additional one to two hours. Following a bronchoscopy, a hoarse voice, loss of voice, and/or a sore throat may occur temporarily. Additional potential complications (although rare) include a pneumothorax or methemoglobinemia.

The nurse is assessing a client who has a pneumothorax. Which of the following assessment findings should the nurse expect? A. Blood-tinged sputum B. Increased anterior-posterior diameter C. Reduced breath sounds on the affected side D. Auscultation of a loud, rough, grating sound Submit Answer

Explanation Choice C is correct. A pneumothorax has clinical features such as reduced breath sounds on the affected sides, tachypnea, dyspnea, and pleuritic chest pain. Some clients may be asymptomatic, depending on the size of the pneumothorax. Choices A, B, and D are incorrect. A pneumothorax does not have features such as blood-tinged sputum, increased AP diameter (this is a feature in COPD), or auscultation of a loud, rough, grating sound. Additional Info Pneumothorax may be caused by trauma to the chest wall secondary to a traumatic injury. Pneumothorax may also adversely develop during the placement of a subclavian or intrajugular central line. The priority treatment for clients unstable with pneumothorax is the placement of a chest tube.

The nurse is caring for a client with Chronic Obstructive Pulmonary Disease (COPD). What should be the nurse's priority nursing diagnosis for the client? A. Activity intolerance B. Ineffective coping C. Impaired gas exchange D. Self-care deficit Submit Answer

Explanation Choice C is correct. Impaired gas exchange is a priority problem for the client with COPD. The nurse should implement measures that ensure adequate oxygen and carbon dioxide exchange in the client. Choice A is incorrect. There is activity intolerance in a client with COPD; however, it does not take priority over gas exchange, which is impaired in the client due to pathologic changes in his lungs. Choice B is incorrect. Ineffective coping is a problem for the client, but it is a psychosocial problem. Physiological issues should always take priority over psychosocial issues. Choice D is incorrect. A self-care deficit should not be a priority over the gas exchange.

The nurse is caring for a client receiving assist-control (AC) via mechanical ventilation. The nurse understands that this setting is used to do which of the following? A. Allow spontaneous breaths at the client's tidal volume. B. Deliver additional pressure at the end of exhalation. C. Deliver a preset tidal volume during spontaneous breaths. D. Provide inspiratory pressure during ventilations.

Explanation Choice C is correct. Assist-control (AC) is a volume mode on a mechanical ventilator that senses a client's ability for a spontaneous breath. When the client takes a spontaneous breath, it will deliver the tidal volume preset on the ventilator. This is in addition to the client receiving the ventilated breaths preset in the rate. For example, if the client is at a preset rate of 12 and taking 4 spontaneous breaths, each breath of the 16 will receive 515 mL of gas (the tidal volume preset). Choices A, B, and D are incorrect. AC differs from SIMV (synchronized intermittent mandatory ventilation); in SIMV, the client may trigger a spontaneous breath and their own tidal volume. Pressure added at the end of exhalation to prevent alveoli collapse would be a description of PEEP (positive end-expiratory pressure). Inspiratory pressure during ventilation would be a description appropriate for pressure support ventilation. Additional Info When caring for a client on a ventilator, you should be familiar with the following settings: Mode (Volume [SIMV, A/C] or Pressure [PSV]) Rate (Number of breaths per minute) Tidal volume (the amount of gas delivered to the client) Fraction of inspired oxygen (FiO2 - the percentage of oxygen given per breath) PEEP (pressure added at exhalation to keep the small airways open and mitigate atelectasis) Pressure support (PS - provides added pressure when the client takes a spontaneous breath)

The nurse has provided medication instruction to a client who has been prescribed formoterol. Which of the following statements would indicate a correct understanding of the teaching? A. "I will take this medication if I experience shortness of breath." B. "I will need to rinse my mouth out after using this medication." C. "This medication may make it hard for me to fall asleep." D. "I should take this medication two hours before I go exercise."

Explanation Choice C is correct. Beta-adrenergic agonists may cause a client to develop insomnia because the medication has the propensity to activate the client and their adrenergic receptors. Drugs in this class (albuterol, salmeterol) share the same effect, insomnia. Choices A, B, and D are incorrect. Formoterol is a long-acting beta-agonist and is indicated in the management of chronic respiratory illnesses. This medication should not be used in acute dyspnea. The client is not required to rinse their mouth out following the use of this medication. This is appropriate instruction for inhaled corticosteroids such as fluticasone. This medication is given on a scheduled basis, and a medication such as montelukast is given two hours prior to exercise to prevent exercise-induced asthma. Additional Info Long-acting beta-agonists (LABAs) are indicated in the maintenance treatment of asthma. The client should be taught that this medication is not indicated for acute exacerbations. Adverse reactions of LABAs include tachycardia, palpitations, and angina. Last Updated - 21, Nov 2022

The nurse is auscultating lung sounds on a client. The nurse understands that these lung sounds are best heard A. right second intercostal space. B. midclavicular line, in the fifth intercostal space. C. posteriorly, between the scapula. D. over the trachea.

Explanation Choice C is correct. Bronchovesicular sounds occur over major bronchi where there are fewer alveoli. They are moderate in pitch and amplitude and are normally equal during inspiration and expiration. Posteriorly, bronchovesicular breath sounds can be auscultated between the scapulae. Choices A, B, and D are incorrect. The right second intercostal space is where the nurse may auscultate the aortic valve. The midclavicular line in the fifth intercostal space is the apex of the heart where the apical pulse may be obtained, and any adventitious sounds such as S3 or S4 may be auscultated. Auscultating lung sounds over the trachea is where the nurse may listen to bronchial breath sounds. These breath sounds are loud and high-pitched with hollow quality.

A hospice nurse is taking care of a client with pancreatic cancer. The client's breathing becomes progressively deeper with periods of apnea. What is this breathing pattern called? A. Kussmaul B. Ataxic C. Cheyne-Stokes D. Biot's Submit Answer

Explanation Choice C is correct. Cheyne-Stokes respirations are characterized by irregular respirations with periods of crescendo and decrescendo, with periods of apnea. It usually indicates brain dysfunction. Choice A is incorrect. Kussmaul respiration is characterized as deep, labored breathing associated with diabetic ketoacidosis. Choice B is incorrect. Ataxic breathing is similar to agonal breathing, characterized by completely irregular breathing with irregular periods of apnea. There is no pattern with this type of breathing, which is the case with Cheyne-Stokes respiration. Choice D is incorrect. Biot's breathing is similar to Cheyne-Stokes in that it is pattern-like. Biot's breathing can be characterized by short, shallow breaths followed by irregular periods of apnea.

The nurse is preparing a client for a bronchoscopy the following day. All of the following are appropriate interventions, except: A. Educate the client that he might be experiencing a sore throat after the procedure. B. Tell the client that he will lie on his back or sit up for 30 to 60 minutes for the procedure, depending on his provider's preference and the procedure's specific goal(s). C. Educate the client that he will be able to eat immediately after the procedure is completed. D. Inform the client he must not eat or drink for six hours before the test. Submit Answer

Explanation Choice C is correct. Following the procedure, the client will be kept NPO (nothing by mouth) until the client's cough and gag reflex return. Upon the return of those protective reflexes, the client will be provided ice chips and small sips of water before slowly progressing into a regular diet to minimize the risk of aspiration. Choice A is incorrect. Due to the possibility of trauma to the larynx and/or pharynx by the bronchoscope, a temporary sore throat is a possible effect following a bronchoscopy which occurs in some clients. Choice B is incorrect. A diagnostic bronchoscopy lasts approximately 30 to 60 minutes (without the pre- or post-op times), during which the client is placed in a sitting or supine position. Choice D is incorrect. The client is kept NPO a minimum of six hours before the procedure to decrease the risk of aspiration. Learning Objective Correlate oral intake immediately after a bronchoscopy with an increased risk of aspiration. Additional Info Bronchoscopy should only be performed by a pulmonologist or trained surgeon in an operating room or intensive care unit. Patients should remain nothing by mouth for a minimum of six hours before a bronchoscopy and have IV access, intermittent blood pressure monitoring, continuous pulse oximetry, cardiac monitoring, and supplemental oxygen for the procedure. Patients usually receive conscious sedation with short-acting benzodiazepines, opioids, or both before the procedure to decrease anxiety, discomfort, and cough. General anesthesia is also commonly used. The pharynx and vocal cords are anesthetized with nebulized or aerosolized lidocaine.

The nurse is assessing a client with a chest tube for a pneumothorax. The nurse assesses a crackling sensation beneath the fingertips around the chest tube insertion site. The nurse should take which action? A. Document the finding as normal B. Clamp the chest tube C. Notify the primary healthcare provider (PHCP) D. Apply nasal cannula oxygen

Explanation Choice C is correct. Notifying the PHCP is essential because this assessment indicates crepitus which is air trapped in and under the skin, known as subcutaneous emphysema. The PHCP needs to be notified because this is a complication, and measures such as increasing the suction on the chest tube need to be considered. Choices A, B, and D are incorrect. These actions are not appropriate. Documenting the finding as normal is inappropriate because this is a complication of chest tube therapy. Chest tubes should rarely be clamped; if they are clamped, they should be for a very brief period. This is not an oxygen issue; thus, applying nasal cannula oxygen is inappropriate and unnecessary. Additional Info Subcutaneous emphysema is the presence of air in the skin's tissue layers and is usually seen as skin swelling around the puncture site. Air in these tissues makes a crackling sound when pressure is applied to it. The presence of subcutaneous emphysema may indicate a persistent air leak caused by a puncture that tears the pleura.

The nurse is helping a client with a chest tube ambulate to the bathroom. The client turns suddenly and the chest tube becomes dislodged. What is the priority action for the nurse to take? A. Immediately re-insert the tube and call for help. B. Place your hand over the chest tube site and yell for help. C. Place a sterile dressing taped on three sides over the chest tube site and call for help. D. Monitor the patient's vital signs while he finishes ambulating to the bathroom and then call for help.

Explanation Choice C is correct. Placing a sterile dressing that is taped on three sides over the chest tube site and calling for help would be the appropriate actions. By placing a sterile dressing over the site the nurse follows infection prevention. By taping the dressing on three sides the dressing will cover the site; this will prevent a tension pneumothorax by allowing exhaled air to escape the dressing. The nurse should then immediately call for help. Choice A is incorrect. Under no condition should the nurse ever reinsert the chest tube. This is not in the scope of practice of the nurse and it is not safe to insert a dirty item into the chest cavity of the client. Choice B is incorrect. It is not appropriate for the nurse to place their hand over the chest tube site. The chest tube site leads directly into the thoracic cavity of the client, so placing a hand over it is an infection risk. Choice D is incorrect. It is not appropriate to allow the client to finish ambulating to the bathroom and simply monitor the vital signs. Chest tube dislodgement is an emergency that requires immediate action. NCSBN Client Need: Topic: Reduction of Risk Potential; Subtopic: Potential for Complications of Diagnostic Tests/Treatments/Procedures, Respiratory Last Updated - 10, Feb 2022

The nurse is caring for a client with acute onset pulmonary edema who is experiencing a desaturating with a Spo2 level of 72% on room air. Which oxygen delivery device is most appropriate for the nurse to apply to the client? A. Simple facemask B. Nasal cannula C. Nonrebreather mask D. Partial rebreather mask Submit Answer

Explanation Choice C is correct. Pulmonary edema is a medical emergency that may cause a client to develop respiratory arrest. Immediate treatment measures for pulmonary edema include providing oxygen at its highest concentration via a nonrebreather mask. This device may deliver up to 95% FiO2. If this is not effective, the provider may consider BiPAP, CPAP, or intubation with mechanical ventilation. Choices A, B, and D are incorrect. A simple facemask, nasal cannula, and a partial rebreather mask simply do not deliver the amount of oxygen required for this emergency. NCLEX Category: Physiological Adaptation Activity Statement: Medical Emergencies Question type: Knowledge/comprehension Additional Info Nasal cannula oxygen can deliver 24%-44% FiO2 at 1-6 liters/minute. A simple face mask can deliver 40%-60% FiO2 at 5-8 L/min; the flow rate must be set at least at 5 L/min to flush the mask of carbon dioxide. A partial rebreather can deliver 60%-75% at 6-11 L/min, a liter flow rate high enough to maintain a reservoir bag two-thirds full during inspiration and expiration. A nonrebreather mask can deliver 80%-95% FiO2 at a liter flow high enough to maintain a reservoir bag two-thirds full.

The nurse is caring for a client who is receiving newly prescribed salmeterol. Which of the following prescribed medications requires notification to the primary healthcare provider (PHCP)? A. Lithium B. Captopril C. Labetalol D. Clonidine

Explanation Choice C is correct. Salmeterol is a long-acting beta-agonist indicated in the maintenance treatment of chronic respiratory illnesses. This medication causes bronchodilation by innervating the adrenergic receptors. Blocking these receptors by beta-adrenergic blockers is contraindicated because it may lead to bronchospasm. Thus, labetalol would be contraindicated because of its risk of this adverse effect. Choices A, B, and D are incorrect. Lithium, a mood stabilizer used in bipolar disorder, captopril, an ACE inhibitor used in heart failure, and clonidine, used in hypertension, is not contraindicated while a client receives salmeterol. NCLEX Category: Pharmacological and Parenteral Therapies Activity Statement: Adverse Effects/Contraindications/Side Effects/Interactions Question type: Analysis Additional Info Long-acting beta-agonists (LABAs) are indicated in the maintenance treatment of asthma. The client should be taught that this medication is not indicated for acute exacerbations. Contraindications for salmeterol include beta-blockers which may cause bronchospasm. Last Updated - 28, Apr 2022

The ER nurse assesses a patient for tactile fremitus. Which would be the correct way to assess for this abnormal finding? A. Percuss the apices in the supraclavicular areas. B. Instruct the patient to breathe deeply while auscultating both sides of the lungs. C. Ask the patient to say "ninety-nine" while palpating the chest. D. Place the hands along the anterolateral wall with thumbs pointing toward the xiphoid process. Submit Answer

Explanation Choice C is correct. Tactile (vocal) fremitus describes the vibrations that can be palpated through the chest wall during speech. To assess for tactile fremitus, the nurse would place hands over the lung apices in the supraclavicular areas and palpate from one side to the other while the patient repeats "ninety-nine" to compare vibrations. If vibrations are uneven, it may indicate pneumothorax, inflamed lung tissue, or fluid build-up. Choice A is incorrect. This would be the correct hand placement, however, assessing tactile fremitus involves no percussing. Choice B is incorrect. Assessing for tactile fremitus involves palpating for vibrations during speech, not auscultating. Choice D is incorrect. This would be the correct process for assessing the costal angle and symmetric chest expansion, not to assess for tactile fremitus. Last Updated - 12, Feb 2022

According to the Global Initiative for Chronic Obstructive Lung Disease (GOLD) report. the leading risk factor for the development of chronic obstructive pulmonary disease (COPD) is: A. Genetics B. Gender C. Cigarette smoking D. Socioeconomic factors Submit Answer

Explanation Choice C is correct. The GOLD report identifies cigarette smoking as the leading risk factor for the development of COPD. Any or all of the others may also be contributing factors. "Never smokers" may develop COPD, but they typically do not have an increased risk of lung cancer or cardiovascular problems compared to smokers with COPD. Choice A is incorrect. Genetics does seem to play a part in the development of COPD. Choice B is incorrect. Gender may play a role in the development of COPD since it may influence occupation choice which may increase exposure to environmental toxins. Choice D is incorrect. Low birth weight influenced by socioeconomic status may hurt lung development. NCSBN Client Need Topic: Reduction of Risk Potential, Sub-topic: Potential for Alterations in Body Systems, Respiratory

The nurse is planning to assist a respiratory therapist in performing a chest physiotherapy procedure. Which of the following is the initial action by the nurse before the process? A. Place a gown or fabric between the hands or percussion device and the client's skin B. Walk with the patient for a few laps around the unit to aid in percussion C. Administer a prescribed bronchodilator D. Call the physician to confirm x-ray results Submit Answer

Explanation Choice C is correct. The nurse should make sure that the patient receives a prescribed bronchodilator about 15 minutes before their chest physiotherapy procedure. Chest physiotherapy is used to loosen secretions trapped in the lungs. When administered before this procedure, a bronchodilator helps to dilate the bronchioles and liquify secretions. Choice A is incorrect. A gown or piece of fabric should be placed between the hands or percussion device right before the procedure. However, this should be done just before the process. Another option (administering bronchodilator 15 minutes prior) exists in the choices and is the initial action. Choice B is incorrect. Walking with the patient before the procedure is not necessary before chest physiotherapy. Choice D is incorrect. Calling the physician to confirm the x-ray results is not necessary at this time and does not alter the plan for chest physiotherapy. NCSBN client need Topic: Physiological Integrity, Reduction of Risk Potential Last Updated - 26, Sep 2021

A client with lung cancer recently had a left lower lobe removal. Which postoperative intervention will be performed as a priority in the care of this patient? A. Tracheostomy B. Mediastinal tube C. Incentive spirometer D. Closed chest drainage system Submit Answer

Explanation Choice D is correct. A patient with a recent lower lobe lung removal will have a chest tube drainage system to collect the blood and drainage and to prevent it from accumulating in the chest. Choice A is incorrect. The patient will likely not have a tracheostomy. Choice B is incorrect. A mediastinal tube is unlikely to be prescribed for this client. Choice C is incorrect. The patient may use an incentive spirometer during their recovery; it is not a priority nursing action. NCSBN Client need Topic: Reduction of Risk Potential / Potential for Complications of Diagnostics Tests, Treatments or Procedures Last Updated - 13, Jan 2022

The nurse is providing discharge instructions to a client with a tracheostomy. Which of the following instructions should the nurse include? A. You may use lemon glycerin swabs for mouth care. B. Remove the old tracheostomy ties before applying the new ties. C. You may use warm tap water to clean the inner cannula. D. Wear a shower shield over the tracheostomy when bathing. Submit Answer

Explanation Choice D is correct. A shower shield should be placed over the tracheostomy when the client bathes. This would prevent water from entering the tracheostomy and potentially lead to pneumonia. Choices A, B, and C are incorrect. Lemon glycerin swabs are not recommended for mouth care because they are drying and may increase the number of bacteria in the mouth. This may lead to dental caries or pneumonia. Removing the old tracheostomy ties before applying the new ones is dangerous because this may allow decannulation if the client coughs. If the inner cannula is not disposable, it should not be washed with tap water. Rather, ½ strength hydrogen peroxide and sterile saline should be used to clean the device. NCLEX Category: Physiological Adaptation Activity Statement: Illness management Question type: Application Additional Info Home care measures for a tracheostomy should be thoroughly taught to the client and the caregiver. These topics should include tracheostomy care, emergency procedures, and oxygen safety. A shower shield is utilized while the client is bathing and comes in various materials. The nurse should emphasize using this shield to decrease the risk of pneumonia.

The nurse is assessing a client with carbon monoxide (CO) poisoning. Which of the following would be an expected finding? A. Decreased pulse oximetry (SpO2) B. Hyperarousal C. Bradycardia D. Headache Submit Answer

Explanation Choice D is correct. CO poisoning is odorless, colorless, and tasteless. This potentially lethal poison initially causes clients to develop symptoms such as headache, reduced visual acuity, and slight breathlessness. As the CO level increases, it causes hypotension, confusion, and vertigo and then progresses to death. Choices A, C, and D are incorrect. A common misconception about CO poisoning is that it causes a decrease in SpO2. This is not accurate because pulse oximetry does not differentiate COHb from oxyhemoglobin. It is entirely plausible and likely that the client has a normal SpO2. CO poisoning would not cause hyperarousal. Instead, it would cause the client to experience dizziness and lethargy. As a compensatory mechanism from the falling cardiac output, tachycardia is commonly seen - not bradycardia. Additional Info ✓ Carbon monoxide binds to a red blood cell approximately 200x more than oxygen ✓ Carbon monoxide poisoning may occur from smoke inhalation from fires, poorly functioning heating systems, and motorboat and motor vehicle exhaust exposure in a closed setting ✓ Manifestations include headache, dizziness, weakness, malaise, altered mental status, and visual changes ✓ This poisoning is tasteless, odorless, and colorless ✓ Treatment includes removing the client from the source of the poison, putting them outside, calling EMS, and administering 100% high-flow oxygen Last Updated - 04, Feb 2023

The nurse is caring for a client with carbon monoxide (CO) poisoning. The nurse anticipates administering oxygen via A. nasal cannula. B. venturi mask. C. simple mask. D. nonrebreather mask. Submit Answer

Explanation Choice D is correct. CO poisoning requires aggressive oxygenation at a FiO2 of 100%. A nonrebreather is the only delivery device to provide this high oxygen level beyond mechanical ventilation. Choices A, B, and C are incorrect. CO poisoning requiring 100% oxygenation would exclude these devices. A nasal cannula can deliver 1-6 L/min for oxygen concentration (FiO2) of 24% (at 1 L/min) to 44% (at 6 L/min). Venturi mask can deliver 4-10 L/min oxygen flow for FiO2 of 24%-55%. A simple face mask can deliver 5-8 L/min oxygen flow for FiO2 of 40%-60%. Additional Info ✓ Carbon monoxide binds to a red blood cell approximately 200x more than oxygen ✓ Carbon monoxide poisoning may occur from smoke inhalation from fires, poorly functioning heating systems, and motorboat and motor vehicle exhaust exposure in a closed setting ✓ Manifestations include headache, dizziness, weakness, malaise, altered mental status, and visual changes ✓ This poisoning is tasteless, odorless, and colorless ✓ Treatment includes removing the client from the source of the poison, putting them outside, calling EMS, and administering 100% high-flow oxygen Last Updated - 13, Feb 2023

The RN is preparing a patient for a pneumonectomy. What teaching should the nurse discuss with the patient? A. Instruct patient to lie on their non-operative side following the procedure. B. Expect the remaining lung to return to normal function within 2-6 hours. C. Advise the patient to avoid coughing, assure that the nurse will use wall suction to clear secretions. D. Keep the head of the bed elevated at a 30-45 degree angle. Submit Answer

Explanation Choice D is correct. Keeping the head of the bed between 30-45 degrees will minimize respiratory efforts and facilitate recovery post pneumonectomy. Choice A is incorrect. The patient would be instructed to lie on the back or operative side only to prevent leaking of fluid into the operative site and to allow full expansion of the remaining lung. Choice B is incorrect. The remaining lung will require 2-4 days to adjust to increased blood flow. Choice C is incorrect. Deep breathing, coughing, and splinting are encouraged during the post-op period to promote the expansion of the lung. Wall suction is contraindicated after pneumonectomy. NCSBN Client Need Topic: Respiratory, Subtopic: Potential for complications from surgical procedures, non-pharmacological comfort interventions Last Updated - 13, Feb 2022

The Registered Nurse is preparing a patient for a pneumonectomy. What teaching should the nurse discuss with the patient? A. Instruct patient to lie on the non-operative side following the procedure. B. Expect the remaining lung to return to normal function within 2-6 hours. C. Advise the patient to avoid coughing and make sure the nurse will use wall suction to clear secretions. D. Keep head of bed elevated at 30-45 degree angle post-procedure. Submit Answer

Explanation Choice D is correct. Keeping the head of the bed between 30-45 degrees will minimize respiratory efforts and facilitate recovery post-pneumonectomy. This intervention will also prevent post-pneumonectomy pulmonary edema. The patient should lie on the operative side and should have the head of the bed raised to 45 degrees as soon as awake. These positions minimize the gravitational effect on capillary pressure in the remaining lung. Choice A is incorrect. Lying on the non-operative side will increase the risk of pulmonary edema and therefore, should be avoided. The patient would be instructed to lie on the back or operative side only to prevent leaking of fluid into the operative side (pulmonary edema) and to allow full expansion of the remaining lung. Choice B is incorrect. The remaining lung will require 2-4 days to adjust to increased blood flow. Choice C is incorrect. Deep breathing, coughing, and splinting are encouraged during the post-op period to promote the expansion of the lung. Wall suction is contraindicated after pneumonectomy. Last Updated - 15, Nov 2022

A nurse is assigned to care for a client with pneumonia. The health care provider (HCP) has prescribed "amoxicillin/clavulanate potassium 500 mg PO TID" for treatment. The medication is available in 250 mg tablets. After calculating the dosing, the nurse finds that the client will receive two tablets every 8 hours. When verifying the client's understanding, the client states, "I should take the medication with food, and if I feel fine before I finish the prescription, I can stop it and save the rest of the pills for the next time I get pneumonia." After rechecking the dosage calculation, the nurse decides to do which of the following? A. Hold medication administration and clarify the dosage with the HCP B. Administer one tablet only C. Administer the medication as prescribed and monitor the client's cardiac function D. Reinforce client teaching and administer the medication as prescribed Submit Answer

Explanation Choice D is correct. Reinforcement of client teaching is needed. The nurse must re-educate the client on the importance of completing the entire course of this medication regardless of the perceived symptom improvement to ensure the bacterial organism is eradicated, thereby decreasing the likelihood that bacteria will develop resistance and not be treatable by antimicrobials. Choice A is incorrect. There is no need to hold the administration of this medication to clarify with the HCP, as this medication and dosage are both indicated and safe. Choice B is incorrect. Administering one tablet would be a medication error. Choice C is incorrect. No monitoring is required unless a client receives long-term antibiotic therapy with this medication. In the event long-term use was needed, renal, hepatic, and hematopoietic (i.e., not cardiac) studies would be intermittently assessed. Learning Objective Recognize the need to reinforce the importance of completing the entire course of antibiotics to a client to ensure the bacteria has been eradicated and to decrease the likelihood of antibiotic-resistant bacteria developing. Additional Info Assess for allergies to penicillins or cephalosporins, as this medication would be contraindicated in these clients. Each dose should be taken with a meal or snack to reduce the possibility of gastrointestinal upset.

Which of the following medication classes are considered quick-relief or rescue medications for a child having an acute asthma attack? Select all that apply. A. Corticosteroids B. Leukotriene modifiers C. Short-acting beta-2 agonists D. Anticholinergics Submit Answer

Explanation Choices C and D are correct. Short-acting beta-2 agonists are "rescue" medications used for bronchodilation in an acute asthma attack. Examples include albuterol and salbutamol. A "rescue" medication is the one that can provide relief even after bronchospasm is triggered (Choice C). Anticholinergics are rescue medications used for the relief of acute bronchospasm. Examples include ipratropium and tiotropium (Choice D). Choice A is incorrect. Corticosteroids are long term control medications used to reduce inflammation. They are not immediately useful as "rescue" medications but are useful in long term management of persistent asthma. Choice B is incorrect. Leukotriene modifiers are long term control medications used to prevent bronchospasm and inflammatory cell infiltration. They are often used as prophylactic agents before a triggering event, for example, in exercise-induced asthma. They are not useful as a "rescue" once bronchospasm occurs. For example, montelukast is indicated to be used "as needed" before exercising in patients who do not require a daily bronchodilator. Montelukast is taken at least two hours before the initiation of exercise. NCSBN Client Need: Topic: Physiological Integrity, Subtopic: Pharmacological and Parenteral Therapies

The nurse is caring for a client following a bronchoscopy. Which assessment finding requires follow-up? A. The client coughs when having small sips of water B. The client complains of a sore throat C. The client expectorates green sputum D. The client complains that he cannot catch his breath

Explanation Choice D is correct. The client states that they feel short of breath, indicating possible respiratory distress. A common complication of bronchoscopy is pneumothorax. The nurse should immediately assess the client for other pneumothorax symptoms (decreased breath sounds on the affected side, tachypnea, tachycardia) and initiate appropriate interventions. Choices A, B, and C are incorrect. The client's coughing indicates that their cough and gag reflexes are returning. These protective reflexes prevent the aspiration of food, water, and other foreign bodies into the lungs. Due to the possibility of trauma to the larynx and/or pharynx by the bronchoscope, a temporary sore throat is a possible side effect following a bronchoscopy present in some patients. The client's expectorating green sputum should not be a cause for alarm to the nurse. Learning Objective Correlate a post-bronchoscopy client complaint reporting an inability to catch one's breath to the possibility of a pneumothorax and escalate appropriately. Additional Info A bronchoscopy is a procedure used to examine the lungs and collect tissue samples for biopsy. Although rare, a pneumothorax does occur in 2-5% of post-transbronchial biopsy clients. Manifestations of this complication may include dyspnea, tachypnea, tachycardia, and coughing. The nurse should not feed the client until the gag reflex has returned.

The nurse is assessing a client immediately following a thoracentesis. The nurse understands that the most common complication following this procedure is a A. Pleural effusion B. Pneumonia C. Pulmonary embolism D. Pneumothorax Submit Answer

Explanation Choice D is correct. The most common complication associated with thoracentesis is a pneumothorax. The nurse should assess the client for this adverse reaction which includes the client experiencing tachypnea, coughing, decreased or absent lung sounds on the affected side, and decreased blood oxygen levels. Choices A, B, and C. These complications are not directly linked with thoracentesis as pneumothorax is much more likely to occur. Additional Info Thoracentesis is the needle aspiration of pleural fluid or air from the pleural space for diagnostic or management purposes. This test can be performed at the bedside and typically involves using ultrasound to guide the needle. Nursing considerations for this procedure involve witnessing the informed consent, positioning the client over a bedside table, and supporting the client during the procedure. The provider will insert a needle (after the skin has been anesthetized) attached to a syringe and will slowly aspirate fluid. This fluid may be sent for laboratory analysis. A sterile pressure dressing will be applied, and a follow-up chest x-ray will be performed. The most common complication following this procedure is a pneumothorax. Last Updated - 15, Jul 2022

A client admitted for a pneumothorax three days ago accidentally pulled out his chest tube. Which action should be the nurse's initial intervention? A. Arrange for a chest x-ray. B. Reinsert the tube herself. C. Notify the physician. D. Place a vaseline gauze over the incision site.

Explanation Choice D is correct. The nurse should cover the incision site with an occlusive dressing whenever a chest tube is pulled out. This action places a seal over the site. The nurse should then notify the physician regarding the incident. Choice A is incorrect. A chest x-ray might be ordered by the physician to check for lung expansion. However, this is not the initial action for the nurse. Choice B is incorrect. The nurse is not allowed to insert a chest tube into the client. Choice C is incorrect. The nurse must place an occlusive dressing first before notifying the physician. Last Updated - 13, Oct 2021

You are caring for an 80-year-old woman with a long-standing history of asthma. You are preparing to give a dose of theophylline to the patient. You know that the most critical sign to assess before giving this dose is: A. Temperature B. Blood Pressure C. Urinary Output D. Pulse

Explanation Choice D is correct. The nurse should evaluate the character of the pulse since one of the toxic effects of theophylline is cardiac arrhythmias. If the pulse rate is significantly increased or erratic, it may alert the nurse regarding a potential arrhythmia from theophylline drug toxicity. Severe adverse events, including arrhythmias, seizures/ status epilepticus, nausea with vomiting, and hypotension, usually occur when the theophylline is at a toxic level in the body (drug toxicity). If such signs are detected, the nurse should hold the next dose of theophylline and immediately notify the healthcare provider. Choices A, B, and C are incorrect. Theophylline does not typically change the patient's temperature (choice A). Theophylline can affect the patient's blood pressure and urinary output, but these effects are less common. The pulse should be assessed first because arrhythmias are more common with theophylline toxicity than hypotension (choice B). Learning Objective Upon assessment, recognize the warning signs of theophylline drug toxicity. Additional Info Although theophylline does not cure asthma, it can help to control symptoms if taken regularly. Because of its narrow therapeutic window, it has the potential to cause drug toxicity. Nurses should be aware of the common side effects versus the toxic effects so theophylline can be held if there are any warning signs of toxicity upon the assessment. Theophylline should be given with a full glass of water on an empty stomach.

A semiconscious client in the postanesthesia care unit (PACU) is experiencing dyspnea. Which action should the nurse perform first? A. Place a pillow under the client's head B. Remove the oropharyngeal airway C. Administer oxygen by mask D. Reposition the client to keep the tongue forward Submit Answer

Explanation Choice D is correct. The tongue can obstruct the airway of a semiconscious client. Repositioning in the side-lying position with the face slightly down will prevent occlusion of the pharynx and allow the drainage of mucus from the mouth. Choice A is incorrect. A pillow under the head increases the risk of aspiration or airway obstruction. Choice B is incorrect. Because the issue is airway obstruction, efforts to promote an open airway are most appropriate. The nurse would want to keep the airway in place. Choice C is incorrect. The issue is airway obstruction, not the percentage of available oxygen. Last Updated - 21, Dec 2021

The registered nurse is asked to assist the physician with removal of a chest tube. Which steps does the nurse anticipate will occur during the procedure? Select all that apply. A. Placing an occlusive dressing over the site. B. Asking the client to bear down as the tube is removed. C. Clamping the chest tube for 30 minutes prior to removal. D. Placing the drainage system near the head of the bed.

Explanation Choices A and B are correct. The nurse anticipates placing an occlusive dressing over the site immediately after the chest tube is removed. It is necessary for this dressing to be airtight to prevent any re-entry of air into the pleural space while the chest tube site is healing (Choice A). The nurse anticipates that the client will be instructed to take a deep breath, exhale, and bear down as the physician quickly removes the chest tube. This helps to ensure no air is inhaled into the pleural space while the chest tube is pulled out and occlusive dressing is placed (Choice B). Choice C is incorrect. It is not typical that the chest tube is clamped prior to removal, so the nurse does not anticipate this. A chest tube is considered ready for removal once the lung has fully re-expanded and there is little to no drainage into the chest tube. Once these criteria are met the physician may decide to remove the tube. Choice D is incorrect. It is not appropriate to place the drainage system near the head of the bed. The drainage system should always be placed below the level of the chest tube site to allow gravity to drain contents into the drainage system. If the nurse placed the chest tube drainage system near the head of the bed, contents could flow back into the chest tube site causing issues such as a pleural effusion. NCSBN Client Need: Topic: Reduction of Risk Potential, Potential for Complications of Diagnostic Tests/Treatments/Procedures; Subtopic: Adult Health - Respiratory Last Updated - 15, Nov 2021

The nurse in the clinic is caring for a 10-year-old with asthma. The child uses an albuterol multi-dose inhaler before engaging in exercise. The nurse should educate the child and parents that potential side effects of this short-acting beta-2 agonist (SABA) are: Select all that apply. A. Tachycardia B. Hypotension C. Headache D. Hypoglycemia Submit Answer

Explanation Choices A and C are correct. According to the National Asthma Education and Prevention Program Expert Panel Report 3, potential side effects of all the SABAs include tachycardia, headache, hypertension, hyperglycemia, tremors, hypokalemia, and increased lactic acid accumulation. The inhaled route is relatively safe since there are few systemic effects from the medication. Choices B and D are incorrect. Hypotension and hypoglycemia are not side effects of SABA use. NCSBN Client Need Topic: Pharmacological and Parenteral Therapies, Sub-topic: Adverse Effects/Contraindications/Side Effects/Interactions; Pharmacology; Respiratory

The nurse is assisting with monitoring a client that has a chest tube and documents the appropriate assessments. Which of these assessments are expected findings? Select all that apply. A. Drainage system at a level below the patient's chest. B. Vigorous bubbling in the water-seal chamber. C. Stable water in the tube of the water-seal chamber during inhalation and exhalation. D. Occlusive dressing over the chest tube. Submit Answer

Explanation Choices A and D are correct. It is expected that the drainage system will be at a level below the client's chest. This is what allows gravity to help drain fluid from the pleural space. If the drainage system was above the client's chest, the chest tube would not work properly (Choice A). An occlusive dressing placed over the chest tube is appropriate. This is important to ensure that air does not enter the pleural space causing a pneumothorax. The nurse should check the dressing to ensure that it is airtight (Choice D). Choice B is incorrect. Gentle bubbling in the water chamber is an appropriate finding, but the bubbling should not be vigorous. Gentle bubbling indicates that air is draining from the client, but if vigorous or excessive bubbling is noted, there may be an air leak, which will need to be addressed quickly. Choice C is incorrect. It is not expected for the water in the tube of the water-seal chamber to be stable during inhalation and exhalation. The water in the tube of the water-seal chamber should fluctuate during inhalation and exhalation. If it does not, the chest tube could be occluded, the lung could have re-expanded, or there could be air leaking into the pleural space. The nurse will need to notify the physician of this finding to investigate the cause and take appropriate action. NCSBN Client Need: Topic: Potential for Complications of Diagnostic Test/Treatments/Procedures, Reduction of Risk Potential; Subtopic: Chest tubes, Respiratory

Which of the following should be included when teaching a 65-year-old male client with COPD about exercise? Select all that apply. A. Instruct the client to avoid sudden position changes that may cause dizziness. B. Recommend that the client restrict fluid until after exercising is finished. C. Instruct the client to push a little further beyond his fatigue with each session. D. Instruct the client to avoid exercising in very cold or very hot temperatures. E. Encourage the client to exercise if he feels ill or weak. F. Recommend to consume a high-carb, low protein diet.

Explanation Choices A and D are correct. Teaching points for exercising in a patient with COPD include avoiding sudden position changes that may cause dizziness and avoiding extreme temperatures. Choices B, C, E, and F are incorrect. Older adults should consume a high-protein, high-calcium, and vitamin D-enriched diet ( Choice F). The nurse should also instruct the patient to provide for adequate hydration, limit fatigue by not pushing to the point of exhaustion, and avoid exercise if weak or ill ( Choice E). NCSBN Client Need Topic: Physiological Integrity, Subtopic: Physiological Adaptation

The nurse is reviewing a client's list of medications who has cystic fibrosis. The nurse anticipates a prescription for which medication? Select all that apply. A. Multivitamin B. Aspirin C. Warfarin D. Simvastatin E. Salmeterol Submit Answer

Explanation Choices A and E are correct. Cystic Fibrosis is a multisystem disorder that causes gastrointestinal disturbances such as malabsorption of essential fat-soluble vitamins (A, D, E, and K). A multivitamin is prescribed to help mitigate these vitamin deficiencies. Salmeterol is a long-acting bronchodilator and has utility in cystic fibrosis as the airways may become narrowed or obstructed. Choices B, C, and D are incorrect. These medications are not utilized in cystic fibrosis. Additional Info Cystic fibrosis is a multisystem disorder that has no cure. A well-balanced diet rich in calories, protein, and fat is recommended to help prevent (or treat) the malabsorption associated with CF. Foods rich in sodium are also recommended because of the salt loss through the skin. A multivitamin is commonly prescribed to help mitigate the vitamin deficiencies that may develop.

You are assigned to take care of a client who just underwent a cholecystectomy. Which of the following would decrease the risk of developing atelectasis in this client? Select all that apply. A. Deep inspiration. B. Supine position with the head end of the bed elevated. C. Change position every 2 hours. D. Encourage the patient to cough at least 10 times/hr. Submit Answer

Explanation Choices A, B, C, and D are correct. Atelectasis is defined as the total or partial collapse of the alveoli. This is a common complication in the immediate postoperative period, especially after abdominal surgeries. If atelectasis is not addressed, it may progress to pneumonia. Since alveoli are responsible for gas exchange, alveolar collapse can lead to impaired gas exchange/impaired oxygenation. Post-operatively, the client may not be able to take deep breaths due to pain from the movement of abdominal muscles. This impaired expansion of the alveoli leads to the accumulation of secretions/mucus plug, decreased surfactant, as well as the obstruction of airway and collapse of alveoli. Additional factors that predispose to this may include hypoventilation, sedation, and reduced mobility. When such factors are identified, the nurse should encourage the client to adopt interventions to mitigate those factors and prevent atelectasis. Such interventions include: Encouraging clients to take deep inspirations (Choice A) and use incentive spirometry. An incentive spirometer encourages the client to pursue deep breathing. Deep breathing aids in gas exchange and promotes the full expansion of the alveoli. Keeping the client in the supine position with the head end of the bed elevated (Choice B) or semi-recumbent area (head of the bed raised 30 to 45 degrees). This allows for maximum thoracic expansion by lowering the abdominal pressure on the diaphragm. Encouraging the client to change position at least every 2 hours (Choice C). This increases mobility and allows full chest expansion and increases perfusion to both lungs. Encouraging the client to cough at least ten times per hour (Choice D) when awake. This helps promote alveolar expansion. The above interventions are aimed at preventing atelectasis. However, the nurse should be aware of detecting atelectasis if it did end up happening. Physical exam findings assist in the diagnosis and include fever and decreased breath sounds on the side of atelectasis. In the case of complete atelectasis/collapse, the trachea/mediastinum may be shifted to the same side due to the pull by a collapsed lung. Atelectasis in the postoperative period is referred to as "resorption atelectasis" but the nurse should also be aware of other types in different client scenarios. Once the nurse detects atelectasis, treatment interventions from a nurse's perspective include: Use of incentive spirometry (IS) - IS mimics the natural process of sighing or yawning. It encourages the patient to take slow and deep breaths. The result of this process is decreased pleural pressure, increased lung expansion, and improved gas exchange. Regular repetition of IS can prevent or even reverse atelectasis. Supportive devices to assist with deep coughing. Chest physiotherapy includes tapping on the chest to loosen mucus Mobilizing the patient early, i.e. encouraging sitting up in bed, sitting over the edge of the bed, standing, or assisted ambulation. Postural drainage - to achieve this, the body is positioned with the head lower than the chest to promote gravitational drainage of the mucus from the bottom of the lungs. (Note this position is for treatment of atelectasis and is different from the semi-recumbent area used to prevent atelectasis) Bronchoscopy may be ordered in certain cases by the physician to remove the mucus plug if the patient is not showing improvement despite the above non-invasive measures. Last Updated - 01, Nov 2021

The nurse is caring for a client immediately following an ultrasound-guided thoracentesis. Which client finding requires follow-up? Select all that apply. A. Nagging cough B. Trachea slanted more to the unaffected side C. Rapid heart rate D. Localized discomfort at the needle site E. Crackling sound made at the insertion site when palpated Submit Answer

Explanation Choices A, B, C, and E are correct. Complications following a thoracentesis include pneumothorax, bleeding, and infection. The biggest concern is pneumothorax. Following a thoracentesis, the client should be assessed for increased respiratory rate, elevated heart rate, nagging cough, decreased oxygen saturation, decreased breath sounds, and air that makes a popping sound near the insertion site when palpated. These are complications and warrant an immediate chest radiograph and supportive measures. Choice D is incorrect. Localized pain at the insertion site is expected. Lidocaine is commonly used prior to the insertion of the catheter. However, some clients may have minor discomfort following this procedure. Additional Info ✓ Thoracentesis is the needle aspiration of pleural fluid or air from the pleural space for diagnostic or management purposes. ✓ This test can be performed at the bedside and typically involves using ultrasound to guide the needle. ✓ Nursing considerations for this procedure involve witnessing the informed consent, positioning the client over a bedside table, and supporting the client during the procedure. ✓ The provider will insert a needle (after the skin has been anesthetized) attached to a syringe and slowly aspirate fluid. ✓ This fluid may be sent for laboratory analysis. ✓ A sterile pressure dressing will be applied, and a follow-up chest x-ray may be ordered. ✓ The most common complication following this procedure is pneumothorax. Last Updated - 31, Dec 2022

What findings are expected when assessing a patient with atelectasis? Select all that apply. A. Decreased breath sounds B. Decreased tactile fremitus C. Hyperresonance D. Shortness of breath E. Decreased oxygen saturation Submit Answer

Explanation Choices A, B, D, and E are correct. Incomplete lung expansion or the collapse of alveoli, known as atelectasis, prevents pressure changes and gas exchange by diffusion in the lungs. With atelectasis, lung tissue has collapsed, which leads to less lung mass available for oxygenation. The oxygen saturation is decreased, as well as breath sounds. Additionally, the patient will experience shortness of breath. Since alveoli collapse, there is more open space between the lung tissue and the chest wall. Open space does not transmit sound very well (decreased tactile fremitus). Areas of the lung with atelectasis cannot fulfill the function of respiration. Coughing, chest pain, cyanosis, dyspnea, and tachycardia are common symptoms of atelectasis. Choice C is incorrect. The percussion sound may be dull but not hyper-resonant in atelectasis or consolidation. Learning Objective Recognize the physical findings of atelectasis Last Updated - 13, Feb 2023

The nurse is providing discharge instructions to a client prescribed nasal cannula oxygen. Which of the following instructions should the nurse include? Select all that apply. A. Keep a pulse oximetry device readily available. B. Pad the tubing in areas that put pressure on the skin. C. Have a sign on your door indicating the presence of oxygen. D. I should use the oven and not the stovetop to cook. E. You may apply petroleum jelly to your nares to prevent drying. Submit Answer

Explanation Choices A, B, and C are correct. A pulse oximetry device should be provided to the client, and they should be encouraged to log their oxygen saturations as directed. If the client experiences dyspnea or tachypnea, the client should be instructed to seek medical attention for a level less than 95% (unless otherwise directed). Padding the tubing around pressure ears (back of the ears) is recommended to avoid injury. A sign posted on the door should be visible to alert visitors of the oxygen and extinguish and open flames. Choices D and E are incorrect. Stovetop and oven cooking is highly discouraged as the presence of oxygen may accelerate any fire that may ignite. Rather, if cooking is to be done using heat or flames, another individual should do the cooking, and the oxygen should be greater than six feet away from the flame source. Petroleum jelly should not lubricate the nares as it may be aspirated. Water-soluble jelly is recommended. NCLEX Category: Physiological Adaptation Activity Statement: Illness management Question type: Application Additional Info For a client being discharged with oxygen therapy, important teaching points to emphasize include: Have a pulse oximetry device readily available. Avoiding any open flame or heat. This includes an oven, stovetop, candles, matches, and cigarettes. Flammable products such as alcohol and oil should be avoided. Have working smoke detectors in the home as well as fire extinguishers. Use a water-soluble jelly to lubricate the nasal passages and mouth to prevent drying.

The nurse is conducting a health screening at a local health fair. Which of the following should the nurse recognize as a risk factor for developing pulmonary tuberculosis (TB)? Select all that apply. A. Human Immunodeficiency Virus (HIV) B. Organ transplant C. Chronic corticosteroid use D. Influenza vaccination E. Obesity Submit Answer

Explanation Choices A, B, and C are correct. According to the Centers for Disease Control (CDC), risk factors for TB include immunosuppression, organ transplant, chronic corticosteroid use, substance use, diabetes mellitus, and residing in environments such as nursing homes, prisons, and homeless shelters. Choices D and E are incorrect. Obesity is not a risk factor for TB. Low body weight is a risk factor for TB. Influenza vaccination is irrelevant to pulmonary tuberculosis and does not raise or lessen the risk for infection. Additional Info Pulmonary tuberculosis is a pathogen spread via aerosolized droplets ➢ If TB is suspected, the client should be placed in airborne precautions. This includes a room with negative pressure or a specialized HEPA filter. The client should not be cohorted with another client and the door must remain closed. ➢ Signs and symptoms of TB include pleuritic chest pain, fever, chills, night sweats, lethargy, and bloody sputum. ➢ When an individual is symptomatic with TB, they have a high risk of transmitting it to others. ➢ Once the symptoms recede, the risk of transmission significantly decreases.

The patient is diagnosed with acute pancreatitis. Which preventative intervention should the nurse implement to reduce the patient's risk of developing a respiratory infection? Select all that apply. A. Assist the patient to turn and reposition frequently. B. Document the respiratory rate and oxygen saturation. C. Place the patient in a semi-fowlers position. D. Encourage deep breathing and coughing. Submit Answer

Explanation Choices A, C, and D are correct. Respiratory infections are common in acute pancreatitis due to retroperitoneal fluid pushing the diaphragm upwards and causing the patient to take shallow abdominal breaths. Assisting the patient to change positions frequently, encouraging deep breathing as well as coughing exercises, and positioning patients for maximum chest expansion would all be preventative interventions to reduce the risk of respiratory infection. Choice B is incorrect. The question is looking for preventative actions to reduce the patient's risk of respiratory infection. While documentation would be indicated to recognize any changes or complications, it would not prevent disease. NCSBN Client Need Topic: Infection control, Subtopic: Potential for complications from health alterations, system-specific assessments, alterations in body systems, illness management, pathophysiology Last Updated - 12, Nov 2021

The nurse is caring for a client who had a fenestrated tracheostomy tube placed one week ago. Which statements are true regarding fenestrated tracheostomies? Select all that apply. A. This type of tracheostomy does not require trach care B. The client with a fenestrated tracheostomy can speak C. This is the only type of tracheostomy used with mechanical ventilation D. A fenestrated tracheostomy can be capped if the cuff is deflated

Explanation Choices B and D are correct. B is correct. It is true that clients with a fenestrated tracheostomy can speak. Fenestrated tracheostomy tubes have a small opening in the outer cannula. This allows some air to escape through the larynx, which means that the client will be able to speak with this type of tube. D is correct. A fenestrated tracheostomy can be capped if the cuff is deflated. It is very important to remember to deflate the cuff if capping a fenestrated tracheostomy tube, because if the tube is capped and the cuff is still inflated the client will not be able to breathe at all. Choice A is incorrect. A client with a fenestrated tracheostomy will require the same amount of trach care as other types of tracheostomies. It is very important to keep the tracheostomy site clean to prevent skin breakdown, infections of the stoma, tracheitis, and respiratory infections. Choice C is incorrect. Fenestrated tracheostomy tubes are not the only type of tracheostomy used with mechanical ventilation, there are also non-fenestrated tracheostomy tubes. A fenestrated tube would be used as a client progresses and is being weaned from breathing only via the tracheostomy to some breathing through the nose and mouth. Fenestrated tracheostomy can also be used with mechanical ventilation, but the cuff must be inflated. NCSBN Client Need: Topic: Alterations in Body Systems, Physiological adaptation; Subtopic: Adult Health - Respiratory

The nurse is teaching a patient who is scheduled for a thoracentesis. Which of the following information should the nurse include? Select all that apply. A. "This procedure will require you to receive general anesthesia." B. "You will need to report any shortness of breath following the procedure." C. "You will need to empty your bladder before this procedure." D. "After the procedure, a follow-up chest x-ray will be done." E. "You will need to be on a clear liquid diet one day before the procedure." Submit Answer

Explanation Choices B and D are correct. These two statements should be included in patient education about thoracentesis. A thoracentesis is a procedure indicated for pleural effusions. The client will need to report any dyspnea after the procedure (Choice B). Shortness of breath following the thoracentesis procedure may indicate either iatrogenic pneumothorax or re-expansion pulmonary edema. Pneumothorax is a common complication following thoracentesis (studies report post-thoracentesis pneumothorax rates ranging from 0 to 19%). The nurse should assess the client carefully for any signs of pneumothorax. Symptoms and signs of a pneumothorax include shortness of breath and reduced or absent breath sounds on the affected side. A more severe pneumothorax, such as tension pneumothorax, may present with obstructive shock. A nurse must notify the physician immediately if any of such signs/symptoms were to occur. A chest x-ray (Choice D) must be completed post-procedure to make sure there is no iatrogenic pneumothorax even if the patient did not show any of the above signs or symptoms. Re-expansion pulmonary edema (REPE) is a complication that occurs after rapid re-expansion of a collapsed lung within 1 to 24 hours. It has been reported <1% in most studies are associated with high mortality. The pathophysiologic mechanism of REPE is unknown. Clinical features vary from cough and chest tightness to acute respiratory failure. Treatment is usually supportive and includes continuous non-invasive positive pressure ventilation or mechanical ventilation in severe cases; some patients also require vasopressors, steroids, and diuretics. Choice A is incorrect. Thoracentesis is a bedside procedure and can be completed under local anesthesia. Choice C is incorrect. It would be inappropriate to advise that the client empty their bladder before the procedure. Choice E is incorrect. Finally, a clear liquid diet one day before the procedure would be appropriate for a colonoscopy - not for a Thoracentesis.

Which of the following are not part of the upper respiratory tract? Select all that apply. A. Adenoids B. Trachea C. Sinuses D. Pharynx E. Epiglottis F. Bronchus

Explanation Choices B and F are correct. The trachea and bronchi belong to the lower respiratory tract, not the upper. The respiratory tract is divided into two sections: the upper respiratory tract and the lower respiratory tract. The upper respiratory tract and the mouth function as the entry point of air and food into the body. The nose, mouth, and throat serve as a common channel for air to reach the lungs and food to enter the esophagus and stomach. The upper respiratory tract warms, filters, humidifies, and transports air into the lower respiratory tract. The upper respiratory tract includes the nostrils, nasal cavities, pharynx, epiglottis, and larynx. Pharynx is often referred to as Throat. Larynx ( voice box) is the portion of the airway between the pharynx and the trachea. Larynx is the transition point between upper and lower respiratory tracts. The larynx contains two important parts: the epiglottis and the vocal cords. The lower respiratory tract includes the trachea, bronchi, bronchioles, and lungs. Choices A, C, D, and E are incorrect. All of these answer choices constitute parts of the upper respiratory tract. Adenoids (Choice A) are lymphatic glands located behind the nasal cavity, usually at the nasopharynx (a part of the upper respiratory tract). Adenoids and the tonsils belong to the lymphatic system. They help fight infections. Sinuses (Choice C), throat/pharynx (Choice D), and epiglottis (Choice E) are parts of the upper respiratory tract as well. Knowing the parts of the respiratory system helps the nurse to identify the source/site of the symptoms. Certain serious conditions like epiglottitis in children should be identified right away. The epiglottis is a component of the larynx. It is a small, movable leaf-like structure just above the larynx that serves as a lid, preventing food and drink from entering the airway. If the epiglottis becomes swollen (epiglottitis), it may obstruct the airway. Epiglottitis refers to infection/inflammation of the epiglottis, which may cause stridor, and is characterized by the "four D's" - dysphagia (difficulty swallowing), dysphonia (muffled voice), drooling, and distress. Stridor is a high-pitched sound during breathing. An inspiratory stridor (stridor during breathing in) suggests airway obstruction above the glottis, such as in the case of acute epiglottitis. NCSBN Client Need I Topic: Health Promotion and Maintenance, Subtopic: Nose, Sinuses, Mouth, and Throat; Structure and Function

You are caring for a 25-year-old asthmatic. According to the National Asthma Education Prevention Program Expert Panel Report-3 (NAEPP EPR-3), you know that the following are risks for death due to asthma: Select all that apply. A. Living in a rural area B. Recent withdrawal from corticosteroids C. 3 or more ED visits for asthma in the past year D. Problems with the perception of obstruction of airflow Submit Answer

Explanation Choices B, C, and D are correct. Asthma is a chronic inflammatory airway disease. Chronic inflammation tends to limit airflow, increase respiratory symptoms, and produce hyperresponsive airways. When working with asthma patients, it is essential to know the risk factors for death due to asthma. These risks include previous ICU admission for asthma, two or more hospitalizations for asthma in the past year, three or more ED visits for asthma in the past year, one hospitalization or ED visit for asthma in the past month, difficulty with perception of asthma symptoms (mainly airflow obstruction), inner-city residence, low socioeconomic status, illicit drug use, and comorbid cardiovascular, lung, or chronic psychiatric disease. Asthma patients with any of these risk factors should be monitored very carefully. Choice A is incorrect. Living in a rural area is not a risk for asthma death; instead, the inner-city residence is the risk factor. NCSBN Client Need Topic: Health Promotion and Maintenance, Sub-Topic: Health Promotion/Disease Prevention Last Updated - 06, Oct 2021

The nurse is counseling a client about a metered-dose inhaler. Which of the following statements by the client indicates effective teaching? Select all that apply. A. "I will be careful not to shake the canister before using it." B. "I will inhale the medication through my nose." C. "After I deliver a dose, I will hold my breath for 10 seconds." D. "I will only inhale one spray with one breath." E. "I will exhale completely and then press down on the inhaler to release the medication." Submit Answer

Explanation Choices C, D, and E are correct. For clients with a metered dose inhaler (MDI), after a dose is administered, they should hold their breath for ten seconds to allow for the medication to be dispersed in their lungs. The client should only administer one dose (or press the button once) per breath. Before the client presses the button to administer the dose, the client is instructed to exhale completely and then administer the dose during the next inhalation. Choices A and B are incorrect. Common mistakes clients make when using metered-dose inhalers include failing to shake the canister and inhaling through the nose rather than the mouth. Additional Info The steps for using a metered dose inhaler without a spacer Tilt the client's head back slightly. Have the client open his or her mouth; position the inhaler 1 to 2 inches away from the client's mouth. Some clients may measure this distance as 1 to 2 finger widths for self-administration. This is considered the best way to use the MDI without a spacer. Alternatively, the client may place the inhaler mouthpiece in the mouth with the opening toward the back of the throat. Have the client exhale completely, and then press down once on the inhaler to release the medication; have the client breathe in slowly and deeply for 5 seconds. Have the client hold his or her breath for approximately 10 seconds and then exhale slowly through the nose or pursed lips. Last Updated - 20, Nov 2022

The nurse is developing a plan of care for a client with a wet-suction chest tube prescribed wall suction. Which interventions would be appropriate to include? Select all that apply. A. Apply clamps to the tubing to secure it to the bed. B. Strip the tubing at least once every eight hours. C. Report any bubbling in the suction control chamber. D. Ambulate the client with the device below the insertion site. E. Palpate around the insertion site for any crackles or popping.

Explanation Choices D and E are correct. Ambulation with a chest tube is not contraindicated. If the nurse has an order from the primary healthcare provider (PHCP) and it is safe for the client to ambulate, the nurse should ambulate the client with the device distal to the insertion site. Palpating around the insertion site should be done and any crackles or popping should be reported to the PHCP because that indicates an air leak. Choices A, B, and C are incorrect. The tubing should not be clamped to the bed as this would cause an obstruction. It would be appropriate to keep extra tubing loose on the bed. Stripping the tubing would be inappropriate because it would increase the intrathoracic pressure, counterproductive to chest tube therapy. Continuous bubbling in the suction control chamber is normal because wall suction is prescribed for this client. Additional Info The nurse is responsible for assessing the patency of a chest tube by assessing each chamber. The nurse should always ensure client safety by having the appropriate emergency equipment at the bedside. This equipment includes occlusive sterile gauze, a bottle of sterile water, and clamps.

The nurse is preparing to suction a tracheostomy tube. Place the following actions in the correct order for this procedure: Insert the suction catheter without suctioning to the pre-measured depth. Hyperoxygenate the patient. Replace the cap, mist collar, oxygen mask, or other apparatus to the tracheostomy and monitor the patient to ensure they return to baseline. Apply intermittent suction and rotate the suction catheter as it is removed from the tracheostomy. Perform hand hygiene and identify the patient. Submit Answer

Explanation Correct ordered sequence: E, A, C, B, D First, the nurse will perform hand hygiene and identify the patient. Second, hyper-oxygenate the patient to prepare them for the procedure and prevent desaturations. Third, insert the suction catheter without suctioning to the pre-measured depth. The nurse should be sure to not allow the suction catheter to touch anywhere other than the inside of the tracheostomy. It is recommended to insert the suction catheter with the nondominant hand. Fourth, apply intermittent suction and rotate the suction catheter as you remove it from the tracheostomy. The nurse should ensure the suctioning does not exceed 10 seconds. Lastly fifth, replace the cap, mist collar, oxygen mask, or other apparatus to the tracheostomy and monitor the patient to ensure they return to baseline. NCSBN Client Need Topic: Reduction of Risk Potential; Subtopic: Potential for Complications of Diagnostic Tests/Treatments/Procedures, Respiratory


Kaugnay na mga set ng pag-aaral

APUSH chapter 15-17 college board questions

View Set

Test Out Linux Pro 8.5.6 Practice Questions

View Set

Module 11: Digital Ethics and Lifestyle

View Set

Chapter 10 - Buying, Using, Disposing

View Set

Chapter 15: Innate and Adaptive Immunity Prep U

View Set

Chapter 6 - Ethical and Legal Issues

View Set